Download as pdf or txt
Download as pdf or txt
You are on page 1of 607

Traffic Engineering

Dr. Hana Naghawi


1
Traffic Engineering
Traffic Engineering: Civil engineering branch associated
with planning, monitoring, geometric design and traffic
operations of roads and their networks, so that people and
goods move safely and efficiently

Primary Objective: Safety


Other Objectives:
Speed
Comfort (physical ch of veh and roadway)
Convenience (ease of trips and the ability of the transport
system)
Economical
Environmental friendly
2
Traffic Engineering
Components and Main Area:
 Planning/ improvement
 Geometric Design
 Analysis of traffic characteristics ( speed, delay,
volume, capacity, flow,….)
 Traffic safety and design of control devices (
traffic signs and signal)
 Maintenance
 Management

3
Traffic Engineering
Current Focus:
 Intelligent Transportation System
 Intermodal Approaches
 Access Management
 Congestion Management
 Environmental Impacts
 Traffic Impact Assessment

4
Congestion

5
Traffic Stream
Traffic streams are made up of individual
drivers and vehicles interacting with each
other and with the physical elements of the
roadway and its general environment

6
Elements of Traffic Stream (TS) System
(subsystem)
1- road users ( pedestrian, driver, rider/passenger)
2- vehicles (private and commercial)
3- roadway (streets and highways)
4- traffic control devices
5- the general environment/qualitative element
(weather, lighting, density of development, local
enforcement)

It is important to understand the ch. of each element


and how they interact to provide safe & efficient
transportation
7
Elements of Traffic Stream
Road Users/Drivers
Human beings are complex and have a wide range
of ch that influence the driving task
Factors affecting driving characteristics:
 Age
 Fatigue
 Driving under influence: DUI

Variability
 Response Time
 Walking Speed of Pedestrians
 Gap Size for Crossing 8
Characteristics of Road User/ Drivers
1- Psychological Factors
• Motivation: business, recreational, shopping,
work ….
• Emotional factors (anger, aggressiveness ,… )
• Maturity
2- Physical Factors
• Visual characteristics of drivers: the most
important
• Hearing: warning sounds
• Perception- Reaction Process 9
Visual Factors in Driving

10
Visual Factors in Driving
a) Visual Acuity: the ability to see fine details
clearness of vision
- varies among people
- depends on the light level
- the best vision occurs within a cone of 3⁰, clear
vision occurs within 10⁰ & satisfactory vision
occurs within 20⁰
- traffic signs and marking should fall within the
cone of clear vision
11
Visual Factors in Driving
b) Depth vision: important in judging distance &
speed

c) Peripheral vision: the ability to understand objects


outside the cone of clear vision 120⁰ -180⁰

The peripheral field narrows as speed increases to as


little as 100⁰ at 20mph and 40⁰ at 60mph

12
Visual Factors in Driving
d) Color vision: the ability to differentiate one color
from another
- deficiency- color blindness – not of great
significance- shapes,…
- about 8% of male & 4% of females suffer from some
degree of color blindness

e) Glare vision and recovery:


time needed by a person to recover when moving
from dark to light is about 3 sec and can be 6 sec or
more when moving from light to dark
sunglasses can reduce this time 13
Perception Reaction Time (PRT)

The 2nd critical driver ch. is the PRT


The process though which a driver evaluates and reacts to a
situation
can be divided into 4 sub processes

PRT

Perception/ Emotion/ Reaction/


Identification
Detection Decision Response
14
Perception Reaction Process

• Perception
• Identification

• Emotion
• Reaction (volition)

PIEV
Used for Signal Design and Braking Distance
15
Perception Reaction Time (PRT)
Perception: the driver sees a control device, warning
sign, or an object on the road

Identification: understanding/identify the situation

Emotion: the driver decides what action to take in


response to the situation

Reaction: executing/implementing the decision

16
Perception Reaction Time (PRT)
PRT is an important factor in the determination
of the breaking distance, so it is an important
factor in determining the min sight distance and
the yellow phase at a signalized intersection.
Varies among individuals and may vary for the
same person as the occasion changes
depending on how complicated the situation is,
age, DUI, environmental conditions…..

17
Perception Reaction Time (PRT)
Typical Perception-Reaction
time range is:
0.5 to 7 seconds
Affected by a number of factors.
What are they?

18
Perception-Reaction Time Factors
• Environment:
• Urban vs. Rural
• Night vs. Day
• Wet vs. Dry

• Age

• Physical Condition:
• Fatigue
• DUI (Drugs/Alcohol)

• Distractions
• medical condition
• visual acuity
• complexity of situation (more complex = more time)
• expected versus unexpected situation (traffic light turning red vs. dog darting into
road)
19
Age
• Older drivers
– May perceive something as a hazard but not act quickly enough
– More difficulty seeing, hearing, reacting
– Drive slower
• Younger drivers
– May be able to act quickly but not have experience to recognize
things as a hazard or be able to decide what to do
– Drive faster
– Are unfamiliar with driving experience
– Are easily distracted by conversation and others inside the vehicle
– May be more likely to operate faulty equipment
– Poorly developed risk perception
– Feel unbeatable, the "Superman Syndrome” 20
DUI

• Affects each person differently


• Slows reaction time
• Increases risk taking
• Dulls judgment
• Slows decision-making
• Presents peripheral vision difficulties

21
Weather

• Fog
• Rain
• Ice
• Snow
• affects ability to see (snow, fog)
• changes ability to stop (ice, snow, wet)

22
Perception Reaction Time (PRT)

American Association of State Highway &


Transportation Officials (AASHTO) recommends: 2.5
sec.

It was found that this value (2.5 sec) is insufficient in


complicated situations and might be exceeded by 35%

Reaction distance: dᵣ = ST
where S: speed, T: time
23
Perception Reaction Time (Example)
Example
A driver with a PRT of 2.5 sec. is driving at 65
mph when she observes that an accident has
blocked the road ahead. Determine the distance
the vehicle would move before the driver could
activate the breaks. The vehicle will continue to
move at 65 mph during the PRT of 2.5 sec

24
Perception Reaction Time (Example)
Example
A driver with a PRT of 3 sec. is driving at 100
km/hr when she sees a tree blocking the road
ahead and must stop. Determine the distance
the vehicle would move before the driver could
applies the breaks.

25
Perception/Reaction Applications
• Stopping sight distance
• Passing sight distance
• Placement of signs/traffic control devices
• Design of horizontal/vertical curves

26
Elements of Traffic Stream/Road
Users
Pedestrian Characteristics
• Characteristics similar to driver
• Design of pedestrian facilities
• Signal timing – get peds across during red
phase

27
Pedestrians

• Walking Speed varies between 3 to 8 ft/sec


depending on age, gender,….

• Design value is 4 ft/sec

Used to calculate safe pedestrians


crossing time

28
Elements of Traffic Stream/Vehicle
Characteristics
The criteria of highway G.D is partly based on
the vehicle ch. (superelevation, widening)
We use the design vehicle

Power
Lighting
Performance (acceleration, breaking)
Dimensions (Size & weight)
Wheels conditions
Instruments (seat belt)
29
Vehicle Characteristics
The physical characteristics of the vehicle in use and the anticipated for
future are essential in highway G.D

Dimensions (Size & weight)


G.D (lane & shoulder width, grads,
pavement design

Performance
speed (G.D, S.D)
Power (resistance)
Acceleration
Deceleration

Safety Standards
Instruments (air bags, seat belt)
Lighting (this affects night operation) 30
Vehicle Characteristics/ Braking performance
Braking system
Initial speed
Type & condition of tire
Roadway surface (wet/ dry)
Grade

Braking Distance?

31
Braking performance
Braking Distance dᵦ?

dᵦ = Si² - Sf²
30(f+ G)

Where:
f= Friction Coefficient a/g
AASHTO recommends
a: deceleration rate 11.2 ft/sec²
g: acceleration due to gravity 32.2 ft/sec²
G= Grade; + for upgrade, - for downgrade
Si =initial speed, Sf = final speed (mph) 32
Braking performance
Braking Distance dᵦ?

dᵦ = Si² - Sf²
30(f+ G)

Application:
Highways
Signalized Intersection
Sign Placement
Accident Investigation
33
Braking performance (Example)
Example
A motorist traveling at 65 mph on an
expressway intends to leave the expressway
using an exit ramp with max speed of 35 mph.
at what point on the expressway should the
motorist step on the brakes to reduce her speed
to the max allowable speed on the ramp and
just before entering the ramp, if this section of
the expressway has a downgrade of 3%.
34
Braking performance (Example)

dᵦ = Si² - Sf²
30(f+ G)

f = a/g = 11.2/32.2 = 0.35

d= 65²- 35²_____ = 312.5 ft


30(0.35- 0.03)
The brakes should be applied at least 312.5 ft from
the ramp
35
Braking performance (Example)
Example
Design Speed 70 mph
f= 0.28 (wet conditions)
Find SSD?

SSD = Reaction distance + Braking Distance dᵦ

= 840 ft

36
Timing of Y & all Red (Example)

37
Elements of Traffic Stream/Roadways

Roadways are a major component of the traffic


stream, and the specifics of their design have
significant impact on traffic operations.
There are 2 primary categories of service
provided by roadways and their systems:
•Accessibility
•Mobility

38
Roadways
Accessibility refers to the direct connection to adjacent land
and land users provided by roadways.
This accessibility comes in the form of:
•Curb parking
•Driveway access to and off street parking
•Bus stops
•Taxi stands
•Loading zones
Accessibility allows a driver and/or passenger to depart the
transport vehicles to enter the particular land use in
question

39
Roadways
Mobility refers to the through movement of people, goods,
and vehicles from point A to point B in the system

The essential problem for traffic engineer is that the


specific design aspects that provide for good access tend to
retard through movement or mobility

The 2 major services provided by roadway system are often


in conflict

This leads to the need to develop roadway system in a


hierarchal manner, with various classes of roadways
specifically designed to perform specific function
40
Trip Function
The American Association of State Highway and
Transportation Officials (AASHTO) defines 6 different travel
movements that may be present in a typical trip:
•Main movement
•Transition
•Distribution
•Collection
•Access
•Termination

A typical trip has 2 terminals

41
Roadway Classification
All highway systems involve a hierarchal classification by
mix of access and mobility functions provided
There are 4 major classes of highways that may be
identified:
•Limited access facility
•Arterials
•Collectors
•Local streets

42
Roadway Classification

43
Functional Classification
Limited access facility (100% mobility)
Arterials
•higher mobility
•low degree of access

Collectors
•balance between mobility and access

Locals
•lower mobility
•high degree of access

Source: Safety Effectiveness of


Highway Design Features, Volume I,
Access Control, FHWA, 1992

Access mobility 44
Roadway
R.O.W

What is the basic


element of Roadway????

carriageway Lane

2 lane – each direction

Note: the shoulders and median are not part of


a carriageway
45
Traffic Stream/Roadway
Traffic Stream: a flow of traffic in a particular
direction on a carriageway (driver + vehicle + road)
Traffic Lane: is the basic element of a roadway
It is that portion of the carriageway which is used for
passage of a single stream of vehicles
Lane width?
Min = 2.7m (Max vehicle width is 2.5m we need
2.7m to allow adequate passage …. Low speed)
Acceptable = 3.5m
Max = 3.75m (not recommended)
Standard = 3.65m
46
Traffic Stream/Roadway
If lane width was exceeded than 3.65m or
decreased less than 2.7m, capacity and delay
would be affected …. Side friction

47
Classes of Traffic Stream
Lanes occupied by traffic flow
Single lane Rural areas
Low traffic volume
E.X: Single lane bridge
used in low traffic volume
in a rural area
One-way flow
No overtaking
Speed of the traffic stream is restricted by the
slowest vehicle in the stream 48
Classes of Traffic Stream
Lanes occupied by traffic flow
2- lane stream

One-way
One-way flow
Overtaking is permitted
Speed is not restricted
49
Classes of Traffic Stream
Lanes occupied by traffic flow
2- lane stream

GAP: passing sight distance

2-way, 2- lane
2-way flow
Overtaking depends on passing S.D &
acceptable gap opposing traffic
Speed is not restricted 50
Classes of Traffic Stream
Lanes occupied by traffic flow
3- lane stream

one-way
One way flow
Overtaking
Speed is not restricted

51
Classes of Traffic Stream
Lanes occupied by traffic flow
3- lane stream Highest rate of accidents
(head on type)
Should be controlled by a
signal
Used during peak hours in
highly congested urban
2-way areas
2-way flow
Overtaking
Speed is not restricted 52
Traffic Stream Performance (TSP)
Any factor that affects traffic stream performance
affects capacity as well

Factors affecting TSP:


1- geometric design features (H & V alignment,
grades, climbing lanes, sight distance, number of
lanes, width,…)
2- traffic composition (HGV, LGV, PC, BUS)
3- road furniture (signs, signals, street lighting, traffic
control devices …)

53
Traffic Stream Performance (TSP)

4- type and condition of road pavement (road surface)


5- number and form of intersections
6- attractiveness of the road (sight seeing, terrain,..)
7- general environment (weather)
8- parking (bus stop)

54
Traffic Facility Classification

Traffic stream = drivers + vehicles + roadway


Uninterrupted flow facility( rural area):
In which no fixed element external to traffic stream
such as traffic signal that cause interruption to traffic
flow. Traffic conditions are result vehicles interaction
among each other in the same traffic stream and
between vehicles and the environment (Freeways)

55
Traffic Facility Classification
Interrupted flow facility (urban areas):
In which fixed elements such as signal, yield or stop
signs causes interrupts the traffic stream flow. Flow
on such facilities depends not only on the interaction
between vehicles, but is affected by control devices
(City street & Arterials)

External factors:
•Roadway crossing
•Ped. Crossing
•Parking, bus stop
•Traffic signals and signs 56
Traffic Stream Parameters
Traffic stream parameters fall into 2 categories :
Macroscopic (uninterrupted flow) : describe the
traffic stream as a whole
•Volume or flow rate
•Speed and travel time
•Density
Microscopic (interrupted flow): describe the
behavior of individual vehicles or pairs of vehicles
within the traffic stream
•Speed of individual vehicle
•Spacing
•headway 57
Macroscopic Parameters/Volume or flow rate
Volume or flow rate
•Traffic volume is defined as the number of vehicles passing a
point on a highway or given lane or direction of a highway,
during a specified period of time.
•Unite …. Vehicles but it is often expressed as vehicle per
unite time (veh/day, veh/hr)
•Daily volumes are used for general planning (trends)
•Hourly volumes for (peak hours of the day) for detailed
design
•Rates of flow are generally stated in unites of veh/hr but
represent flows that exist for periods less than an hour

58
Volume or flow rate
Volume or flow rate
A volume of 200 vehicle observed over 15 min period
=
200 * 4 = 800 veh/hr flow rate that exists for the 15
min period

59
Volume
Daily Volumes
There are 4 daily volume parameters that are used in
traffic engineering

•Average annual daily traffic (AADT). The number of


vehicles passing a site in a year divided by 365 days
(veh/day)

•Average annual weekday traffic (AAWT).The number


of vehicles passing a site on weekdays in a year
divided by the number of weekdays (260) (veh/day)
60
Volume or flow rate

•Average daily traffic (ADT). The average 24 hr volume


at a given location over a defined time period less
than a year; common application is to measure an
ADT for each month of the year (veh/day)

•Average weekday traffic (AWT). The average 24 hr


volume weekday volume at a given location over a
defined time period less than a year; common
application is to measure an AWT for each month of
the year (veh/day)
61
Volume or flow rate

62
Volume or flow rate
Daily Volumes are generally not differentiated by direction or
lane but are totals for the entire facility at the designated
location

63
Volume or flow rate

Note that ADTs are significantly higher than AWTs in


each month. This suggests that the facility is serving a
recreational area with traffic strongly peaking on
weekends. Also both AWTs & ADTs are highest during
summer months…..
Thus if a detailed study were needed to provide data
for upgrading of this facility, the period to focus on
would be weekends during summer

64
Volume or flow rate
Hourly volumes
Max volumes occurs during the morning &
evening peak (rush hour)
The single hour of the day that has the highest
hourly volume is referred to as the peak hour
The peak hour volume is stated as a
directional volume (each direction of flow is
counted separately)
Use peak hour volume in the peak direction
for design
65
Volume or flow rate

66
Volume or flow rate
In design, peak hourly volumes are estimated
from projections of AADT
Directional design hourly volumes (DDHV)
DDHV = AADT* K* D
Where:
K: proportion of daily traffic occurring
during peak hour (30th peak hour of
the year)
D: proportion of peak hour traffic
traveling in the peak direction of flow
67
Volume or flow rate

68
Volume

69
Volume or flow rate/Example
Rural highway
AADT (after 20 yr) = 30,000 veh/day

What range of directional design hour volume


might be expected?
DDHV(low) = 30,000* 0.15* 0.65 = 2,925
DDHV(high) = 30,000* 0.25* 0.80 = 6,000

Large range difficulty in projecting future


traffic demand accurately
70
Peak Hour Factor
•The peak hour factor (PHF) is a ratio of the
total hourly traffic volume to the maximum 15-
minute traffic volume within the hour.
•PHF represents the flow variation within an
hour.
PHF = hourly volume = V____
max rate of flow 4*v15

v15: max 15 min volume within the hour, veh


71
Volume or flow rate

72
Volume or flow rate

73
Volume or flow rate

Time interval VOLUME FOR TIME INTERVAL (vehs)


5:00 – 5:15 PM 1,000
5:15 – 5:30 PM 1,100
5:30 – 5:45 PM 1,200
5:45 – 6:00 PM 900
5:00 -6:00 PM 4,200

PHF = 4,200/(4*1,200) = 0.875


Max PHF = 1
When volume of each interval is constant
74
Volume or flow rate
Assume capacity = 4200 veh/hour

Time interval VOLUME FOR TIME DEPARTING QUEUE SIZE AT THE


INTERVAL (vehs) VEHICLES END OF PERIOD (VEH)
5:00 – 5:15 PM 1,000 1050
5:15 – 5:30 PM 1,100 1050
5:30 – 5:45 PM 1,200 1050
5:45 – 6:00 PM 900 1050
5:00 -6:00 PM 4,200

75
Macroscopic Parameters/Speed and Travel Time

Speed and Travel Time

Speed is the 2nd macroscopic parameter


Speed is the rate of motion in distance per
unite time
Travel time is the time taken to traverse a
defined section of the roadway
Speed and travel time are inversely related
S=d
t
76
Speed and Travel Time
In moving traffic stream, each vehicle travels
at a different speed. Thus the TS does not have
a single characteristics value rather a
distribution of individual speeds

The TS taken as a whole can be characterized


using average speed

There are 2 ways in which TS speed can be


computed:
77
Speed
1. Time mean speed (TMS): the average speed of all
vehicles passing a point on a highway or lane
over specified time period

2. Space mean speed (SMS): the average speed of


all vehicles occupying a given section of highway
or lane over a specified time period

78
Speed
Time mean and space mean speed: Know the difference?

Time mean Average speed of all vehicles passing a point


speed (TMS) over some specified time period
TMS = ∑ (d/ti)/n
Space mean Average speed of all vehicles occupying a given
speed (SMS) section over some specified time period or
harmonic mean of individual speeds.
SMS = d/(∑(ti)/n) = nd/∑(ti)

Where:
d: distance traveled
n: # of observed vehicles
ti: `time for vehicle I to traverse a section
79
Illustrative Computation of TMS and SMS

80
Speed types

Do you know the difference among these speeds?


 Average travel speed
 Average running speed
 Operating speed = Max safe speed , without
exceeding the design speed of the highway segment
 Percentile speed = a speed below which the stated
percent of vehicles in the traffic stream travel
81
Macroscopic Parameters/Density
Density & Occupancy
 Density, the 3rd primary measure of TS
characteristics

 Defined as the number of vehs occupying a given


length of highway or lane

 Expressed as (veh/mi/ln) or (veh/mi)

 It is difficult to measure directly, it can be


estimated from speed & flow rate
82
Macroscopic Parameters/Density
Density & Occupancy
 It is computed from speed and flow rate
measurements
 It is the most important parameter of the 3
primary traffic stream parameters because it
relates to demand
 Important measure of the quality of traffic flow

83
Macroscopic Parameters

84
Occupancy as a substitute parameter for density

Density is difficult to measure. So, we use “occupancy” as


a surrogate measure for density. This can be obtained by
traffic detectors of any kind.
Occupancy: the percent of the roadway (in terms of time)
that is covered (occupied) by vehicles.

85
Microscopic Parameters
Spacing & Headway
 Spacing: distance gap between the same physical point of 2
successive vehicles
 Headway: time gap between the same physical point of 2
successive vehicles

86
Microscopic Parameters
 The average spacing in a traffic lane can be directly related to the
density of the lane:
D = 5,280
Sm
Where:
D: density (veh/ml/ln)
Sm: average spacing bet vehs in the lane (ft)

 The average headway in a traffic lane can be directly related to the


flow rate of the lane:
v = 3,600
ha
Where:
v: flow rate (veh/hr/ln)
ha: average headway bet vehs in the lane (sec)
87
Microscopic Parameters
Example:
Traffic in a congested multilane highway,
observed average spacing on a lane is 200 ft
and an average headway of 3.8 sec.
estimate the rate of flow and density?
v = 3,600 = 947 veh/hr/ln
3.8

D = 5,280 = 26.4 veh/ml/ln


200
88
Relationships among v, S & D
The 3 macroscopic measures- flow, speed and density, are
related as follow:
v = S*D
Where:
v: rate of flow (veh/hr) or (veh/hr/ln)
S: SMS (ml/hr)
D: density (veh/ml) or (veh/ml/ln)

SMS & D are measures that refer to specific section of a


lane or hwy, while v is a point measurement (any point within
the section and represent the avg. v on the section)
This equation applies only for uninterrupted flow conditions
89
Relationships among v, S & D
If a freeway lane was observed to have a SMS of 55mi/hr
and a density of 25 veh/mi/ln, estimate flow rate

v = 55 * 25 = 1,375 veh/hr/ln

The relationship is most often used to estimate density

Consider a freeway lane with a measured SMS of 60 mi/hr


and a flow rate of 1,000 veh/hr/ln. estimate the density

D = v = 1,000 = 16.7 veh/mi/ln


S 60
90
Relationships among v, S & D
The previous equation suggest that a given
flow rate could be achieved by infinite # of SMS
& D pairs having the same product

There are additional relationships between


pairs of these variables that restrict the # of
combinations that can & do occur in the field

Next slide illustrates the general form of these


relationships
91
Relationships among v, S & D
The exact shape and calibration of these relationships depends upon
prevailing conditions which vary from location to location and even over
time at the same location

92
A few examples…

93
Fundamental diagram of traffic flow (flow vs.
density)
Optimal flow or Mean free speed, Vf
capacity,Vmax

Optimal speed, uo
Flow (V)

Speed is the slope.


= v/D

Uncongested flow
Congested
flow
Jam density, kj
Density (D) Optimal /critical
density, Do 94
Fundamental diagram of traffic flow (SMS vs.
density & SMS vs. flow)

Vf Vf
Uncongested
flow
SMS

SMS
Congested
flow

0 Dj 0 Vmax
Density Flow

SMS vs. density SMS vs. flow


95
Fundamental diagram of traffic flow and
shock wave
For upstream

V1
Slope gives
Flow (V)

velocity Vw of
V2 shock wave for V1

Work
zone For bottleneck

D1 D2
Density (D) Dj

Queue forms upstream of the bottleneck. So we use the


diagram of the upstream section 96
Relationships among flow rate, speed,
and density
Do you remember
Mean free speed whose flow model is
Optimal used for this?
flow or
Unstable flow area S = Sf –(Sf/Dj)*D
capacity

Optimal speed
Flow (v)

Speed is the slope. S


= v/D

Uncongested flow
Congested
flow
Optimal (critical) Jam density
Density (D) density 97
Relationships among v, S & D
Note that:
A flow rate of 0 veh/hr occurs under 2 very different
conditions:
When there are no veh on the hwy – density is 0
veh/mi – no vehs can be observed passing a point,
under this condition speed is free flow speed
When there are so many vehs on the road that all
motion stops – very high density – jam density
Between these 2 extreme points, the peaking
flow-speed and flow density curves is the max v or
capacity of the roadway
Operation under cap is so unstable 98
Relationships among v, S & D
The solid portion of the curves represents stable flow

The dashed portion of the curves represents unstable or


forced flow (breakdown, queuing,.. v > cap.)

Except for cap. flow any flow rate may exist under 2
conditions:
A condition of relatively high speed and low density (on
stable portion of flow relationship)
A condition of relatively low speed and high density (on
the unstable portion of flow relationship)

99
Relationships among v, S & D
Greenshield conducted the 1st formal studies of
traffic flow and hypothesized that speed – density
relationship was linear

Speed – density relationship is considered to be the


basic behavioral relationship – drivers selecting speed
based on their proximity to other veh (and the
geometric and the general environment of the
roadway)

100
Relationships among v, S & D
Example:-
Consider greenshield’s linear speed – density model
which resulted in the following calibrated
relationship:
S = 55.0 – 0.45D

1. Determine free flow speed


2. Jam density
3. Capacity

101
Capacity & LOS
Capacity is the max hourly rate at which vehs/persons can
reasonably be expected to traverse a point or uniform section
during a given time period under prevailing roadway, traffic
and control conditions

It is essential to:


1. Determine type of hwy facility
2. Estimate the max amount of traffic that can be
accommodated by a facility
3. Provide basis for planning, design & operation of roads
4. For a given # of lanes, it provide a means for determining
the traffic carrying performance (LOS)
102
Capacity & LOS
Prevailing roadway conditions - refer to the
geometric characteristics of the road
1. Type of facility
2. # of lanes
3. Lane & shoulder width
4. Lateral clearance
5. Design speed
6. H & V alignment

103
Capacity & LOS
Prevailing traffic conditions - refer to the characteristics of
TS using the facility
1. Traffic composition
2. Distribution of veh by lane and by direction
3. Speed, vol, density,….

Prevailing control conditions – refer to type & specific


design of control device & traffic regulations
1. Location & timing of traffic signals
2. Turn restrictions

 Under ideal conditions


Freeway cap = 2,000 pcphpl
2- lane 2-way hwy = 2,800 pcph 104
Capacity & LOS
Level of Service (LOS)
A qualitative measure describing the traffic
operational conditions and perception by motorists/
passengers within TS
From A to F, A – best & F – worst
Parameters used to define LOS
1. Speed & delay & travel time
2. Flow interruption
3. Freedom to maneuver
4. Comfort
5. Convenience
6. Safety 105
LOS Flow conditions Freedom to select Level of comfort &
speed & maneuver convenience
A Free flow Extremely high Excellent
B Stable Reasonable Less than A
C Stable Begins to be restricted Decline noticeably
D High density- limit of stable Severely restricted Poor
E near capacity- any increase Low uniform speed Extremely poor
of flow will cause extremely difficult to
operational problem maneuver
106
F Forced flow none Breakdown occurs
Levels of Service
• LOS A
– Free-flow operation

From Highway Capacity Manual, 2000


• LOS B
– Reasonably free flow
– Ability to maneuver is only
slightly restricted
– Effects of minor incidents still
easily absorbed 107
Levels of Service
• LOS C
– Speeds at or near FFS
– Freedom to maneuver is
noticeably restricted
– Queues may form behind any
significant blockage.

From Highway Capacity Manual, 2000


• LOS D
– Speeds decline slightly with
increasing flows
– Density increases more quickly
– Freedom to maneuver is more
noticeably limited
– Minor incidents create queuing 108
Levels of Service
• LOS E
– Operation near or at capacity
– No usable gaps in the traffic
stream
– Operations extremely unstable
– Any disruption causes queuing

From Highway Capacity Manual, 2000


• LOS F
– Breakdown in flow
– Queues form behind
breakdown points
– Demand > capacity 109
Service Flow Rate (SFR)
SFR is the max hourly rate at which vehs/persons can
reasonably be expected to traverse a point or uniform section
during a given time period under prevailing roadway, traffic
and control conditions while maintaining a designated LOS
(cap of each LOS)
Every LOS has SFR except LOS F – no flow

110
Saturation Flow Rate (SF)
Capacity at signalized intersections is based on the concept
of saturation flow and saturation flow rate

Saturation Flow Rate can be defined as the flow of vehicles


per hour that can be accommodated by the lane/ lane group
assuming unlimited green time and infinite queue

Saturation flow rate is given the symbol sf and is expressed


in units of vehicles per hour of effective green time (vphg) for
a given lane group

111
Saturation Flow Rate (SF)
Approach

Opposing
Approach

112
Saturation Flow Rate (SF)
Factors affecting SF:
 Width of approach
SF = 525 w (m)
= 160 w (ft)
where: w is the effective width of approach
SF in pcph

 Gradient
each 1% uphill decreases SF by 3% and each 1% downhill
increases SF by 3%

 Composition of traffic
covert all traffic to pc
113
Saturation Flow Rate (SF)
Factors affecting SF:
 Composition of traffic

Vehicle type PCU


Heavy or medium good veh 1.75
Bus 2.25
Tram 2.50
Light good veh 1.00
Motorcycle 0.33

114
Saturation Flow Rate (SF)
Factors affecting SF:
 Composition of traffic
Passenger Car Unite is essentially the impact that a mode of transport
has on traffic variables (such as headway, speed, density) compared to a
single car. For example, typical values of PCE (or PCU) are:

Vehicle type PCU


Heavy or medium good veh 1.75
Bus 2.25
Tram 2.50
Light good veh 1.00
Motorcycle 0.33
115
Saturation Flow Rate (SF)
Factors affecting SFR:
 Composition of traffic

V= 1,000 vph
500 pc
300 HGV = 300 * 1.75 = 525 pc
200 BUS = 200 * 2.25 = 450 pc
1,000 vph = 500 + 525 + 450 = 1,475 pcph

PCU- factor = PCU value/ veh value = 1475/1000 = 1.475

PCU- factor = 1.10 – 1.35 depending on traffic


composition
116
Saturation Flow Rate (SF)
Factors affecting SF:
 Turning movement
one left turn = 1.75 straight
one right turn (above 10%) = 1.25 straight

1,000 pcph
100 turned left = 100 * 1.75 = 175 straight
600 straight
300 turned right =100 * 1.25 = 375 straight

Turning movement effect factor = 1200/1000 = 1.2

117
Saturation Flow Rate (SF)
Factors affecting SF:
 Site characteristics
• Good
• Average
• Poor

Defined in term of:


• Visibility
• Turning radii
• Pedestrian interference

118
Degree of Saturation (Dsat.)
Used in capacity & operational analysis at signalized
intersections
It is a ratio: arrival flow (demand)/capacity(saturation flow)

Value ranges between 0 & 1


 0 – very low traffic
 1 saturated
 >1 over saturated conditions long queues and
extensive delays
The lower degree of saturation the better LOS
The higher SF the better
The higher capacity the better
119
Capacity, LOS & Dsat.
In the analysis process of cap., LOS, and saturation flow, the
starting point is to set a value for ideal conditions then modify
the values to reflect the prevailing conditions at the site
under study

Ideal conditions = any improvement will not improve


capacity
Ideal conditions for uninterrupted flow facility:
1. 12 ft lane width
2. 6 ft shoulder width
3. 70 mph design speed for multilane hwy
60 mph design speed for 2- lane hwy
4. All PC in the TS
5. No sight restrictions for passing & overtaking 120
Capacity, LOS & Dsat.
Ideal conditions for signalized intersection approach:
1. 12 ft lane width
2. Level grade (usually measured as average slop bet stop
line & a point on the approach 60 m before it)
3. No curb parking on the intersection approach closer to
the stop line than the length of a normal max queue
4. All PC in TS
5. All vehs travelling through the intersection
6. Intersection located in a non CBD area
7. Green signal available all time

121
Capacity, LOS & Dsat.
Factors affecting C, LOS & Dsat.:
Roadway Conditions:

1. Type of facility & roadway conditions (int./unint.)


2. # of lanes
3. Lane & shoulder width
4. Lateral clearance
5. Design speed
6. H & V alignment

122
Capacity, LOS & Dsat.
Factors affecting C, LOS & Dsat.:
- Traffic Conditions – vehicle type:

1. PC: registered as PC/LGV, not more than 4 tires


2. HGV (truck): has more than 4 single tires and used primarily for goods transport
3. Buses: has more than 4 single tires and used primarily to transport people

- Drivers Population
-Commuter Driver: driver that uses the road regularly during weekdays
- Recreational Driver: driver that does not use the road regularly during
weekdays

- Control conditions
1. TCD
2. Marking

123
Capacity, LOS & Dsat.
Terrain Conditions
-Level terrain: any combination of grades H & V alignments
that permits HGV to travel at similar speed to PC

-Rolling terrain: any combination of grades H & V


alignments that causes HGV to travel at slower speed than
PC, but not operating at crawling speed for significant time
& length

-Mountainous terrain: any combination of grades H & V


alignments that causes HGV to operate at crawling speed
for significant time & length
124
Introduction to Traffic Control Devices

125
Traffic Control Devices
Defined in the Federal MUTCD

Markings
Signs
Signals

126
127
Traffic Signs & Marking
• Principles of traffic signs and markings:
Traffic signs and markings are most effective
when they satisfy five basic requirements:
– Fulfill a need;
– Command attention;
– Convey a clear, simple meaning;
– Command respect from road users; and
– Give adequate time for proper response.

128
Traffic Signs
• Definition:
A traffic sign is defined as a device mounted on a fixed or
portable support whereby a specific message is expressed
by means of words or symbols.
The sign is placed for the purpose of regulating, warning, or
guiding vehicular, pedestrian, or bicycle traffic.
• Purpose:
The purpose of traffic control devices, as well as the
principles for their use, is to promote highway safety and
efficiency by providing for the orderly movement of all road
users on streets and highways.
Traffic control devices or their supports shall not allow any
advertising message or any other message that is not
related to traffic control. 129
There are three types (classifications) of
traffic signs
• They are regulatory
• Warning
• and guide.

A. Regulation signs give notice of traffic laws or


regulations.
B. Warning signs give notice of a situation that
might not be readily apparent.
C. Guide signs show route designations,
destinations, directions, distances, services, points
of interest, and other geographical, recreational, or
cultural information. 130
Traffic Signs
Changeable Message Signs
• Changeable message signs are traffic control devices
designed to display variable messages.
• Changeable message signs should not be used to display
information other than regulatory, warning, and guidance
information related to traffic control.
• Changeable message signs, with more sophisticated
technologies, are gaining widespread use to inform road
users of variable situations, particularly along congested
traffic corridors.
• In order to ensure that the above principles are fulfilled
the following aspects should be considered: design;
placement and operation; maintenance; and uniformity.
131
Traffic Signs
Design
Shape, color, size, composition, lighting or retro-reflection should command
attention and convey a simple meaning.

The sign should have:


A. High visibility by day and night; and
B. High legibility (adequately sized letters or symbols, and a short legend for quick
comprehension by a road user approaching a sign).
Shapes
Particular shapes, as shown in the Table below, shall be used exclusively for specific
signs or series of signs.
Use of Sign Shapes
Shape Signs
Octagon * Stop
Equilateral Triangle (1 point down) * Yield
Circle Highway-Rail Grade
Crossing (Advance Warning)
132
Emergency Evacuation Route Marker
Traffic Signs
Color code
The general meanings of colors used in signs are as follows:

Color General Meaning


Yellow Warning
Red Stop or prohibition
Blue Road user services guidance, tourist information, and
evacuation route
Green Indicated movements permitted, direction guidance
Brown Recreational and cultural interest area guidance
Orange Temporary traffic control
Black Regulations
White Regulation
Fluorescent Yellow - Pedestrian warning, bicycle warning, school bus and
Green school warning 133
Traffic Signs
• Size
See the MUTCD for standard sizes of sings.
– Normal sizes of signs are used in urban areas
– Lager signs are used in rural areas.
– Lager signs are also used for added emphasis or to
ensure that they attract motorists attention.

• Illumination
– External light,
– light within or behind the sign,
– Shape, letter or symbol
134
Traffic Signs
• Message:
A. word messages
brief, 3 words maximum on regulatory or warning
signs
B. symbolic
for faster recognition by drivers
• arrows, curve warning, deer crossing
• symbols, should be learned by the code; red circle with a
slash, and DO NOT ENTER SIGN.
• Lettering
Uppercase letters or all regulatory and warning signs
135
Traffic Signs
• Sign location
• On the right hand side of the roadway
• Facing approaching traffic
• Supplementary signs in other locations may be used
• Guide signs are often mounted overhead (on wide and high
speed streets)
• Visible only to the traffic for which they are intended.

– Longitudinal placement
– Lateral placement

136
Traffic Signs
• Longitudinal placement
Must be coordinated with roadside features (guardrails and other signs)
Regulating signs:
normally placed at or near the location where the regulation:
– exists (stop, yield)
– begins (wrong way, do not pass)
– additional signs where a regulation continues over an extended section of the
highway (speed limit)
Warning signs:
– in advance of the hazard
– as a function of approaching traffic speed

Guiding signs:
– In advance of an intersection or junction
– At the point where they apply (street names, kilometer posts)

• Lateral placement
Within the driver's cone of vision 137
Regulating signs

138
Warning signs
•Warning signs are
information to warn
drivers of hazards
•They are usually yellow
and diamond shaped.
There are messages to
help drivers understand
these hazards.
•Although these are not
regulatory signs, not
heeding the warning
may result in an
accident

139
Guide signs
• Guide signs. Guide signs are used to inform
drivers of routes, gas, food, hospitals, rest
areas and others. Guide signs are designated
by color and shape.

140
Blue is motorist services. Gas, food,
lodging, hospitals etc.

141
Brown is for recreational areas.

142
Pavement Markings
• Definition:
Traffic markings are all lines, patterns, symbols,
words, colors, or other devices, except signs and
power-operated traffic control devices, set into the
surface of, applied upon, or attached to the
pavement or curbing and placed for the purpose
of regulating, warning, or guiding traffic.
• Pavement markings, like signs and signals, are
designed to move traffic safely, rapidly, and
efficiently.
• Markings therefore, should communicate a
simple, clear message for all highway users.
143
Pavement Markings
• Marking functions:
Markings may supplement other traffic control
devices or they may be used alone to convey
information that would be difficult to convey using
other devices.
They are specifically used to:
A. Display regulations (no passing zones, curb parking
restrictions)
B. Supplement other devices ( STOP lines, symbol arrows)
C. Guide traffic (lane lines, rout number)
D. Warn traffic (SIGNAL AHEAD, rail-highway crossing
legend) 144
Pavement Markings
• Marking Limitations
Markings have several important limitations.
– May be hidden by:
• other vehicles directly over the markings or by
snow, or by dirt.
– May be worn by sand or gravel.
– May not be visible when wet.
– They wear due to traffic and the environment and must
be maintained or replaced.
– Removal of markings from the pavement is a difficult
task. 145
Pavement Markings
• Color
– White: one-way traffic
– Yellow: two-way traffic
– Blue
• Longitudinal Markings (parallel to the
roadway)
– Broken lines permissive
– Dotted lines path guidance
– Solid lines restrictive
– Double lines maximum restriction
– Width of line indicates degree of emphasis
146
Pavement Markings
 Marking applications
 Before any new highway, paved detour, or temporary
route is opened to traffic, all necessary markings should be in
place.

 Advantages
 Under most highway conditions, markings provide
important information while allowing minimal diversion of
attention from the roadway.

 Pavement markings can enhance roadway definition with


the addition of clear features such as bars, differential surface
profiles, raised pavement markers, or other devices intended
to alert the road user that delineation on the roadway is being
traversed.
147
Pavement Markings / Examples
Yellow Line

Two way traffic pass


with caution

Traffic Flow
Two way traffic no
passing
Traffic Flow

148
Pavement Markings / Examples
White Lines
One-way traffic
Broken - pass with caution

Traffic Flow

Traffic Flow

Stop line

Left turn only - arrow

No passing - solid line


149
Pavement Markings / Examples
Multi-Lane

ONE WAY – pass TWO WAY – no passing over solid


with caution lines

150
Pavement Markings / Examples

Triangles painted on the


pavement show that you must
yield

151
Pavement Markings / Examples
Arrows Indicate the Flow of Traffic

This lane may continue


straight or turn left

This lane must turn left

152
Pavement Markings / Examples

Shared Left Turn Lane


 Traffic from both directions uses
this lane to turn left
 Do not travel more than 150 feet in
this lane

153
Testing Your Knowledge

2. If you are driving in this


lane, you must

a. turn left
b. turn right

154
Pedestrians are still required to obey traffic ligh
Zebra markings are to tell drivers to yield.
155
Handicapped
• Blue pavement markings are used for
handicapped parking.

156
• HOV lanes (High Occupancy Vehicles) are
used in large cities to encourage car
pooling. Generally they are the left lane(s)
marked with a white diamond and require
at least two or three occupants.

157
This double white line has the same
restriction as a double yellow line. Then
what is the difference?

158
What’s the deal here?

159
Intersections

160
Basic principles of intersection
signalization

161
Signal Timing Improvement Practices
• Advantages of traffic signals
– provide for the orderly movement of traffic
– increase the traffic-handling capacity of the intersection
– reduce the frequency of certain types of crashes
• Right-angle
• Left-turn head-on
• Rear end
– coordinated to provide for continuous movement of
traffic at a definite speed along a given route
– interrupt heavy traffic at intervals to permit other traffic,
vehicular or pedestrian, to cross
162
Signal Timing

• The objective of signal timing


– alternate the right of way between traffic streams
• Minimize average delay to all vehicles and
pedestrians,
• Minimize total delay to any single group of vehicles
and pedestrians
• Minimize possibility of crash-producing conflicts

163
Three Colors Used in Traffic Signals
a. Red is always stop. If it is flashing red, you may go as if a
stop sign. A solid red you might be allowed to turn under
certain circumstances.

b. Green means you may go, if safe to do so. There is no


right of way given by a green light.

c. Yellow is used for caution, but it is a clearance light. You


may clear an intersection if the light is yellow, but you are
not allowed to enter an intersection once the light has
changed to yellow.

164
Signalized Intersections

Semi-
Actuated
Actuated

165
Types of Traffic Control
• Pre-timed
– operates on a clock
– same cycle length and split for the designed period
• Actuated
– makes use of detectors (sensors)
• buried in the road
• video detection
• at all approaches
• at some approaches- semi-actuated
– give green time only to the approaches with waiting vehicles
– change the signal as soon as they have been served
– used where traffic volumes are not steady
166
Four critical aspects of signalized intersection operation
1. Discharge headways, saturation flow rates, and lost times
2. Allocation of time and the critical lane concept
3. The concept of left-turn equivalency
4. Delay as a measure of service quality
Cycle length
Phase
Interval
Change interval
All-red interval
(clearance interval)

167
Components of a Signal Cycle
1. Cycle: is one complete rotation through all of indications
provided. In general, every legal vehicular movement
receives a green indication during each cycle, although there
are some exceptions to this rule
2. Cycle length: is the time (in seconds) that it takes to
complete one full cycle of indications. It is given the symbol
“C”.
3. Interval: a period of time during which no indication
changes. It is the smallest unit of time observed within a
signal.
4. Phase: it consists of green interval, plus the change and
clearance intervals that follow it. It is a set of intervals that
allow a designated movement or set of movements to flow
and to be safely stopped before releasing of the conflicting
movements.
168
Types of Intervals
• Change interval: it is the “yellow” indication for a given
movement. It is part of the transition from “green” to “red,” in
which movements about to lose “green” are given a “yellow”
signal, while all other movements have a “red” signal. It is
timed to allow a vehicle that cannot safely stop when the
“green” to enter the intersection legally. It is given the symbol
“yi” for movement “i”.
• Clearance interval: it is also part of the transition from “green”
to “red” for a given set of movements. During the clearance
interval, all movements have a “red” signal. It is timed to
allow a vehicle that legally enters the intersection on “yellow”
to safely cross the intersection before conflicting flows are
released. The clearance interval is given the symbol “ari” “all
red” for movement(s) i.
• Green interval: given the symbol “Gi”
• Red interval: given the symbol “Ri” 169
Signal Timing Design for Isolated
Intersections
• Cycle length
– shortest cycle that will accommodate the demand present
and produce the lowest average delay
• Typical range = 60 seconds to 120 seconds
• Minimize intersection delay
– Delay = actual time – expected time
• Function of each individual vehicles, driver behavior, etc
– Types of delay
• Travel time delay- hard to measure at an intersection
• Stopped time delay- physical counting and analysis
170
Signal operation modes and left-turn treatments
Left-turn treatments:
Permitted left turns
Protected left turns
Protected/permitted (compound) or permitted/protected left
turns
Factors affecting the permitted LT movement
• LT flow rate
• Opposing flow rate
• Number of opposing lanes
• Whether LTs flow from an exclusive LT lane or from a shared lane
• Details of the signal timing 171
Four basic mechanisms for building an analytic model or description
of a signalized intersection
Discharge headways at a signalized intersection
The “critical lane” and “time budget” concepts
The effects of LT vehicles
Delay and other MOEs (like queue size and the number of stops)

MIDDLEBELT ROAD
Total N
Approach
Volume =
1,197 (vph)

 Traffic Volume
– Approach volume
FIVE MILE ROAD 86
111 84
1002 349
Total Approach

– Turning counts Total Approach 129


89
Volume = 524 vph

Volume = 674
– Classification counts vph 451

94
64
996
73

• Trucks Total
Approach

• Buses
Volume =
1,133 vph

172
Total Intersection Volume = 3,528 vph
Discharge headways, saturation flow, lost times, and
capacity

Effective
green

Saturation flow rate

3600 g i  Gi  Yi  t L
s Start-up lost time
Yi  yi  ari
h
t L  l1  l2
l1   (i )
Saturated headway

l 2  y  ar  e
T  l1  nh gi
ci  si
C
Capacity Cycle length
173
173
The fundamental element of a signalized intersection is the
periodic stopping and restarting of the traffic stream. When
the light turns GREEN, there is a queue of stored vehicles
that were stopped during the preceding RED phase, waiting
to be discharged. As the queue of vehicles moves, headway
measurements are taken as follow
The first headway is the time lapse between the initiation of the
GREEN signal and the time that the front wheels of the first vehicle
cross the stop line.
The second headway is the time lapse between the time that the
first vehicle’s front wheels cross the stop line and the time that the
second vehicle’s front wheels cross the stop line.
Subsequent headways are similarly measured.
Only headways through the last vehicle in queue (at the initiation
of the GREEN light) are considered to be operating under
“saturated” con’ditions. 174
The saturation flow rate is the capacity of the approach lane or lanes if
they were available for use all of the time (i.e., if the signal were always
GREEN).
3600 where: s = saturation flow rate, vehicles per hour of green
s per lane (veh/hg/ln)
h h = saturation headway, seconds/vehicle (s/veh)

The additional time involved in each initial headways (above and


beyond “h” seconds) is noted by the symbol Δi (for headway i). These
additional times are added, and are referred to as the startup lost time:

l1   (i ) where: l1 = start-up lost time, s/phase


Δi = incremental headway (above “h” seconds) for vehicle i, s

amount of GREEN time required to discharge a queue of “n” vehicles


as: where: T = GREEN time required to move queue of “n”
T  l1  nh lvehicles
1
through a signalized intersection, s
= start-up lost time, s/phase
n = number of vehicles in queue
175
h = saturation headway, s/veh
The lost time associated with stopping the queue at the end of the GREEN signal. This time is
more difficult to observe in the field, as it requires that the standing queue of vehicles
be large enough to consume all of the GREEN time provided. In such a situation, the clearance
lost time l 2 is defined as the time interval between the last vehicle’s front wheels crossing the
stop line, and the initiation of the GREEN for the next phase. The clearance lost time
occurs each time a flow of vehicles is stopped.

If the start-up lost time occurs each time a queue starts to move and the clearance lost time
occurs each time the flow of vehicles stops, then for each GREEN phase:
t L  l1  l2
The concept of lost times leads to the concept of effective green time.
g i  Gi  Yi  t L
Yi  yi  ari
t L  l1  l2
where: gi = effective green time for
movement(s) i, s
Gi = actual green time for movement(s) i, s
Yi = sum of yellow and all red intervals for
yz = yellow interval for movement(s) i, s
ari = all-red interval for movement(s) i, s
tLi = total lost time for movement(s) i, s
176
movement(s) i, s
Capacity of an Intersection
The saturation flow rate(s) represents the capacity of an intersection
lane or lane group assuming that the light is always GREEN.
The portion of real time that is effective green is defined by the
“green ratio,” the ratio of the effective green time to the cycle length of
the signal - (g/C).
The capacity of an intersection may then be computed as:
where: ci = capacity of lane or lane group i, veh/h
gi si = saturation flow rate for lane or lane group
ci  si
C gi = effective green time for lane or lane group
C = signal cycle length, s

The “critical-lane,’ concept involves the identification of specific lane


movements that will control the timing of a given signal phase.
Each phase has one and only one critical lane (volume). If you have a
2-phase signal, then you have two critical lanes.
177
Example:
Consider the given movement at a signalized intersection with the
following known characteristics:
Cycle length, C = 60 s
Green time, G = 27 s
Yellow plus all-red time, Y = 3 s
Saturation headway, h = 2.4 s/veh
Start-up lost time = 2.0 s
Clearance lost time = 1.0 s
For these characteristics, what is the capacity (per lane) for this
movement?

178
S= 3600/h = 3600/2.4 = 1,500 veh/hg/ln
gi  Gi  Yi  t L
g = 27 + 3 - 3 =27s
c = s * (g/c) = 1,500" (27/60) = 675 veh/h/ln
The two results are, as expected, the same. Capacity is found by isolating the effective green
time available to the subject movements and by assuming that this time is used at the
saturation flow rate (or headway).

179
Finding an Appropriate Cycle Length

Desirable cycle length, incorporating PHF and the desired level of v/c

Nt L where: Cdes = desirable cycle length, s


Cmin 
 Vc  PHF = peak hour factor
1   v/c = desired volume to capacity ratio
 3600 / h  N = number of phases in the cycle
Nt L
Cdes  Vc = maximum sum of critical lane volumes, veh/h
Vc
1
PHF (v / c)(3600 / h)

Doesn’t this look like the Webster model?

1.5L  5
C0  
1   Yi
i 1

180
Traffic Engineering Studies

181
Traffic Engineering Studies
• Traffic studies may be grouped into
three main categories:
– (1) Inventories,
– (2) Administrative studies, and
– (3) Dynamic studies.

182
Traffic Engineering Studies
(1) Inventories:

An inventory is defined as an information system (or data


base) of what is there

An inventory is needed to provide information on both


current demand and supply

provide a list or graphic display of existing information,


such as:
 street widths,
 parking spaces,
 transit routes,
 traffic regulations. 183
Traffic Engineering Studies
(2) Administrative studies
• use existing engineering records, available in
government agencies and departments.
• include the results of surveys, which may
involve:
– field measurements and/or
– aerial photography.

184
Traffic Engineering Studies
(3) Dynamic traffic studies
• involve the collection of data under operational
conditions and
• include studies of:
– speed,
– traffic volume,
– travel time and delay,
– parking, and
– crashes.
• They are described in detail in this chapter.

185
Traffic Engineering Studies
• SPOT SPEED STUDIES
• Spot speed is defined as the average speed of
vehicles passing a point on a highway.
– Spot speed studies are conducted to estimate the
distribution of speeds of vehicles in a traffic stream at a
particular location on a highway.
– carried out by recording the speeds of a sample of vehicles
at a specified location.
• Used to:
– Establish parameters for
– traffic operation and control, such as:
• speed zones,
• speed limits (85th-percentile speed) 186
Traffic Engineering Studies
• Locations for Spot Speed Studies
– Represent different traffic conditions on a highway
for basic data collection.
– Mid-blocks of urban highways and straight, level
sections of rural highways for speed trend
analyses.
– Any location may be used for solution of a specific
traffic engineering problem.

187
Traffic Engineering Studies
 Spot speed should be selected to achieve the following:
 Unbiased data
 Drivers be unaware
 Spot speed studies are conducted when traffic is free-flowing,
 not conducted when volumes are in excess of 750-1,000
veh/h/ln on freeways or 500 veh/h/ln on other types of
uninterrupted flow facilities.
 during off-peak hours.
 typically:
• the duration is at least 1 hour and
• the sample size is at least 30 vehicles.

188
Traffic Engineering Studies
• Sample Size for Spot Speed Studies
– The larger the sample size, will give an
estimated mean within acceptable error limits.
• There are several key statistics that are used to
describe spot speed distributions:
• Average Speed
• Median Speed
• Modal Speed
• The ith-percentile Spot Speed
• Pace
• Standard Deviation of Speeds

189
• Average or time mean speed-the average speed of all
vehicles passing the study location during the period of the
study.
• Standard deviation of speeds is the average difference
between observed speeds and the time mean speed during
the period of the study.
• 85th percentile speed: the speed below which 85% of the
vehicles travel.
• Median:-the speed that equally divides the distribution of
spot speeds; 50% of observed speeds are higher than the
median; 50% of observed speeds are lower than the
median.
• Pace: a 10-mi/h increment in speeds that encompasses the
highest proportion of observed speeds(as compared with
any other 10-mi/h increment). 190
The results of spot speed studies are used for many different
purposes by traffic engineers, including:
• Establishing the effectiveness of new or existing speed limits or
enforcement practices.
• Determining appropriate speed limits for application.
• Establishing speed trends to assess the effectiveness of speed
limits and enforcement.
• Specific design applications determining appropriate sight
distances, relationships between speed and highway alignment,
and speed performance with respect to steepness and length of
grades.
• Specific control applications for the timing of “yellow” and “all
red” intervals for traffic signals, proper placement of signs, and
development of appropriate signal progressions.
• Investigation of high-accident locations at which speed is
suspected to be a contributing cause to the accident
experience. 191
Traffic Engineering Studies
• Methods for Conducting Spot Speed
Studies
– manual and automatic
– manual method is seldom used
– automatic devices
1. road detectors
2. radar-based
3. the principles of electronics.

192
Traffic Engineering Studies
• Road Detectors
– road tubes & loops to collect data on speeds &
volume at the same time
– Advantage:
• Human errors are considerably reduced
– Disadvantages:
• expensive
• may, affect driver behavior,

193
Traffic Engineering Studies
• Radar-Based Traffic Sensors
• Electronic-Principle Detectors
– traffic characteristics, such as speed, volume,
queues, and headways are computed.
– Using video image processing

194
Traffic Engineering Studies
• Presentation and Analysis of Spot Speed
Data
– Statistical methods
– Analyzing data
– frequency histogram
– cumulative frequency distribution curve

195
Traffic Engineering Studies
• Example
• Determining Speed Characteristics from a
Set of Speed Data.
Table 4.2 shows the data collected on a rural
highway in Virginia during a speed study. Develop
the frequency histogram and the frequency
distribution of the data and determine:

196
Traffic Engineering Studies
1. The arithmetic mean speed
2. The standard deviation
3. The median speed
4. The pace
5. The mode or modal speed
6. The 85th-percentile speed

197
Traffic Engineering Studies

Table 4.2 Speed Data Obtained on a Rural Highway 198


• The speeds range from 34.8 to 65.0 mph, giving a speed range of
30.2.
• A frequency distribution table can then be prepared, as shown in
Table 4.3.

Table 4.3
Frequency
Distribution
Table for Set
of Speed
Data

199
Traffic Engineering Studies

Figure 4.4 Histogram of Observed Vehicles' Speeds 200


Traffic Engineering Studies

Figure 4.5 Frequency Distribution 201


Traffic Engineering Studies

Figure 4.6 Cumulative Distribution 202


Traffic Engineering Studies
• The arithmetic mean speed = 4260/86 = 49.5 mph
• The standard deviation = 3632/85 = ±6.5 mph
• The median speed/the 50th-percentile speed = 49 mph
• 85th-percentile speed is 54 mph
• The pace is 45 mph to 55mph
• The mode = 50 mph

203
Traffic Engineering Studies
• VOLUME STUDIES
The most fundamental measurement in traffic
engineering is counting-counting vehicles,
passengers, and/or people.
1. Average Annual Daily Traffic (AADT)
2. Average Daily Traffic (ADT)
3. Peak Hour Volume (PHV)
4. Vehicle Classification (VC)
5. Vehicle Miles of Travel (VMT)

204
Traffic Engineering Studies
• Methods of Conducting Volume Counts
– Manual Method
– Automatic Method

205
Traffic Engineering Studies
• Types of Volume Counts
– Depending on the anticipated use of the data to
be collected.
• Intersection Counts
– vehicle classifications,
– through movements,
– turning movements.
Types of Volume Counts
• Pedestrian Volume Counts
• Periodic Volume Counts (AADT)
206
Traffic Engineering Studies
• Traffic Volume Data
Presentation
– Traffic Flow Maps:
volume of traffic on
each route is
represented by the
width of a band.
Figure 4.13 shows a typical
traffic flow map.

Figure 4.13 Example of a Traffic Flow Map 207


Traffic Engineering Studies
Intersection Summary Sheets:

Figure 4.14 shows a typical


intersection summary
sheet.

Figure 4.14 Intersection Summary Sheet 208


Traffic Engineering Studies
Time-Based Distribution Charts:
see Figure 4.15

Daily
variations:
see Figure
4.15b

Figure 4.15 Traffic Volumes on an Urban Highway (A&B) 209


Traffic Engineering Studies
Hourly
variations in
traffic
volume:

Figure 4.15 Traffic Volumes on an Urban Highway (C) 210


Traffic Engineering Studies
Summary Tables:
PHV, Vehicle Classification (VC), and ADT.
See Table 4.4

Table 4.4 Summary of Traffic Volume Data for a Highway Section

211
Traffic Engineering Studies
• Adjustment of Periodic Counts
• Expansion Factors from Continuous Count Stations.
– Hourly expansion factors (HEFs) are determined by
the formula

These factors are used to expand counts of duration


shorter than 24 hr to 24 hr volumes by multiplying
the hourly volume for each hour during the count
period by the HEF for that hour and fining the mean
of these products
212
Traffic Engineering Studies
– Daily expansion factors (DEFs) are computed as

These factors are used to determine weekly volumes from


counts of 24 hr by multiplying the 24 hr volume by
DEF
– Monthly expansion factors (MEFs) are computed as

The AADT for a given year may be obtained from ADT


for a given month by multiplying this volume by
the MEF 213
Traffic Engineering Studies
Table 4.5 Hourly Expansion Factors for a Rural Primary Road

214
Traffic Engineering Studies
Table 4.6 Daily Expansion Factors for a Rural Primary Road

215
Traffic Engineering Studies
Table 4.7 Monthly Expansion Factors for a Rural Primary Road

216
Traffic Engineering Studies

217
Traffic Engineering Studies

218
Traffic Engineering Studies
• TRAVEL TIME AND DELAY STUDIES
– Travel time: time required to travel from one
point to another on a given route.
– the locations, durations, and causes of delays.
– good indication of the level of service
– identifying problem locations,

219
Traffic Engineering Studies
• To identify problem locations on facilities by
virtue of high travel times and/or delay.

• To measure arterial level of service, based on


average travel speeds and travel times.

• To provide necessary input to traffic assignment


models, which focus on link travel time as a key
determinant of route selection.

220
Traffic Engineering Studies
• Definition of Terms Related to Time and Delay Studies
1. Travel time: time taken by a vehicle to traverse a given
section of a highway.
2. Running time: time a vehicle is actually in motion
3. Delay time lost due to causes beyond the control of the
driver.
4. Operational delay: delay caused by the impedance of
other traffic.
5. Stopped-time delay: the part of delay during which the
vehicle is at rest .
6. Fixed delay: caused by control devices such as traffic
signals, regardless of the traffic volume
7. Travel-time delay: difference between the actual travel
time and the travel time obtained by assuming that a
vehicle traverses at an average speed equal to that for an
uncongested traffic flow
221
Field Study Techniques

• Because significant lengths of roadway are


involved, it is difficult to remotely observe vehicles
as they progress through the study section.

• conducting travel time involve driving test cars


through the study section.

• Recording elapsed times through the section.


Traffic Engineering Studies
Table 4.8 Speed and Delay Information

226
Traffic Engineering Studies
Moving-Vehicle Technique
(moving observer):
– the observer makes a round trip on a test
section Figure 4.16,
– The observer starts at section X-X, drives the
car eastward to section Y-Y,
– turns the vehicle around
– drives westward to section X-X again

227
Traffic Engineering Studies

Figure 4.16 Test Site for Moving-Vehicle Method 228


Traffic Engineering Studies
• following data are collected as
– The time it takes to travel east from X-X to Y-Y
(Te), in minutes
– The time it takes to travel west from Y-Y to X-X
(Tw), in minutes
– The number of vehicles traveling west in the
opposite lane while the test car is traveling
east (Ne)

229
Traffic Engineering Studies

Moving-Vehicle Technique.
– The number of vehicles that overtake the test
car while it is traveling west from Y-Y to X-X,
that is, traveling in the westbound direction
(Ow)
– The number of vehicles that the test car
passes while it is traveling west from Y-Y to X-
X, that is, traveling in the westbound direction
(Pw)

230
Traffic Engineering Studies
Moving-Vehicle Technique.
• The volume (Vw) in the westbound
direction can then be obtained from the
expression:

231
Traffic Engineering Studies
• where (Ne Ow Pw) is the number of
vehicles traveling westward that cross the
line X-X during the time (TeTw).
• Similarly, the average travel time in the
westbound direction is obtained from

232
Traffic Engineering Studies

233
Traffic Engineering Studies

234
Traffic Engineering Studies

235
Traffic Engineering Studies

236
Traffic Engineering Studies

237
Notes:
• The floating-car and average-car techniques result
in estimates of the average travel time through
the section.

• The floating-car technique is generally applied only


on two-lane highways, where passing is rare and the
number of passings can be counted and balanced
relatively easily.

• On a multilane freeway, such a driving technique


would be difficult at best, So we use average-car
technique in this case .
Traffic Engineering Studies
• Methods Not Requiring a Test Vehicle
• License-Plate Observations: observers at
the beginning and end of the test section.
• Each observer records the last three or
four digits of the license plate of each car
that passes, together with the time at
which the car passes.

239
Traffic Engineering Studies
• in the office by matching the times of arrival
at the beginning and end of the test section
for each license plate recorded.
• difference between these times is the
traveling time of each vehicle.
• average of these is the average traveling
time on the test section.

240
Types of Volume Counts

Screenlines, cutline, and cordon surveys

These are surveys undertaken at boundaries or along


imaginary lines
These surveys involve count of all crossing
The surveys may involve interviews of a sample of
vehicle drivers and/or passengers
Some surveys may be done using license plat
techniques

241
Methods of Data Collection
Screenlines, cutline, and cordon surveys
Cordon: closed circle around the study area
Cordon survey
around the CBD
CBD area

Screenline: a line that goes through the study area


screenline that
CBD goes through
the CBD area

Cutline: just a line that you choose in the study area


Cutline that
CBD goes through
the CBD area
242
Traffic Engineering Studies

Figure 4.12 Example of Station Locations for a Cordon Count 243


Traffic Engineering Studies
• a sample size of 50 matched license plates.
• Interviews: obtaining information from
people who drive on the study site regarding
their travel times, experience of delays,
requires the cooperation of the people.

244
Traffic Engineering Studies
• ITS Advanced Technologies:
– Advanced technologies
– Cell phones
– GPS satellite system
– technology is used to determine average speeds
and travel times along highways

245
Traffic Engineering Studies
PARKING STUDIES
• facilities for different type of development (airport,
mall, residential, industrial)
• Any vehicle traveling on a highway will at one time
or another be parked for short time or long time,
depending on the reason for parking

246
Traffic Engineering Studies
• Providing adequate parking space to meet
the demand for parking in the Central
Business District (CBD)
• This problem usually faces a city traffic
engineer.
• solution is not simple
• Parking studies are therefore used to
determine the demand for and the supply
of parking facilities in an area
247
Parking Types
 On-Street Parking:
It’s provided on one or both sides of the street These parking can be:
• Unrestricted parking:
(if the time of parking is unlimited and parking is free)
• Restricted parking :
(if parking is limited to specific times of the day)
– provided to free or not free
– provided for specific purpose (bus stop, loading barking)
 Off-Street Parking :
can be privately or publicly owned.
• Self-parking garages:
require that drivers park their own automobiles
• Attendant-parking garages:
maintain personnel to park the automobiles 248
Definitions used in parking studies:
 A space-hour : defines the use of a single parking space for a
period of 1 hour.
 Parking volume: is the total number of vehicles that park in a
study area during a Specific length of time
 Parking accumulation: is the number of parked vehicles in a study
area at any specified time.
 The parking load: is the area under the accumulation curve
between two specific times. It is usually given as the number of
space-hours used during the specified period of time.
 Parking duration: is the length of time a vehicle is parked at a
parking bay.
As the average parking duration increases, the parking supply
decreases.
 Parking turnover: is the rate of use of a parking space. It is
obtained by dividing the parking volume for a specified period by
the number of parking spaces.
If the turnover rate increases, it is an indicator for higher parker’s
number at a lower parking duration. 249
Traffic Engineering Studies
Methodology of Parking Studies
• Inventory of Existing Parking Facilities
– detailed listing of the location and all other
relevant characteristics of each legal parking
facility, private and public.
– The study area includes both on- and off-street
facilities.

250
Traffic Engineering Studies
• Type and number of parking spaces at each parking
facility
• Times of operation and limit on duration of parking,
if any
• Type of ownership (private or public)
• Parking fees, method of collection
• Restrictions
• Other restrictions, loading and unloading zones, bus
stops, taxi ranks
• Permanency
– The inventory should be updated at regular intervals of
about four to five years. 251
Traffic Engineering Studies
Methodology of Parking Studies
• Collection of Parking Data
– Accumulation:
• by checking the amount of parking during
regular intervals on different days of the
week.
• Carried out on an hourly or 2-hour basis
• used to determine hourly variations of
parking and peak periods of parking demand.
252
Traffic Engineering Studies
• Collection of Parking Data
– Turnover and Duration:
• collecting data on a sample of parking spaces in a
given block.
• recording the license plate of the vehicle parked on
each parking space in the sample at the ends of fixed
intervals during the study period.
• The length of the fixed intervals depends on the
maximum permissible duration.

253
Traffic Engineering Studies
– Turnover and Duration:
• For example, if the maximum permissible duration of
parking at a curb face is 1 hour, a suitable interval is
every 20 minutes.
• If the permissible duration is 2 hours, checking every
30 minutes would be appropriate. Turnover is then
obtained from the equation

254
Traffic Engineering Studies

Figure 4.17 Parking Accumulation at a Parking Lot 255


Traffic Engineering Studies
– Parking Demand
• by interviewing drivers at the various parking facilities
• Interview all drivers using the parking facilities on a
typical weekday between 8:00 a.m. and 10:00 p.m.
• Information include (1) trip origin, (2) purpose of trip,
(3) driver’s destination after parking.
• the location of the parking facility, times of arrival and
departure, vehicle type.

257
Traffic Engineering Studies
– Parking Demand
• Parking interviews also can be carried out using the
postcard technique,
• about 30 to 50 percent of the cards distributed are
returned.

258
Traffic Engineering Studies

263
Traffic Engineering Studies

264
Traffic Engineering Studies

265
Geometric Design
266
Geometric Design
The geometric design of roads is the branch of highway engineering
concerned with the positioning of the physical elements of the roadway
according to standards and constraints. The basic objectives in geometric
design are to optimize efficiency and safety while minimizing cost and
environmental damage

The highway design process requires knowledge of most sub


disciplines of civil eng., including:
Traffic & transportation Eng.
Geotechnical Eng.
Material Eng.
Structural Eng.
Hydraulic Eng.
Surveying

267
Geometric Design
Geometric roadway design can be broken into three main parts:
alignment, profile, and cross-section. Combined, they provide a three-
dimensional layout for a roadway

The alignment is the route of the road, defined as a series of horizontal


tangents and curves

The profile is the vertical aspect of the road, including crest and sag
curves, and the straight grade lines connecting them

The cross section shows the position and number of vehicle and
bicycle lanes and sidewalks, along with their cross slope or banking.
Cross sections also show drainage features, pavement structure and other
items outside the category of geometric design

268
Highway Classification
Classification of roads is necessary for
communication among engineers, administrators, and
the general public
Classification schemes applies for different
purposes:
 By design type/function
Grouping of roads by character of service that they
provide
•Used for transportation planning and design
•The most widely used (Arterial, Collector, Local)

Different function - different characteristics


269
Highway Classification
By route numbering – traffic operation
state – primary
- secondary

A – primary state road, B- secondary state road


A45 is more important than A456
A456 is more important than B1563
A45

270
B1563
Highway Classification
Administrative classification – to identify the level of
government responsibility – method of financing
state/ federal aid – primary
- secondary

271
Functional Classification
Classify the road by function (LOS)

Two important consideration to be taken into account where


functional classification:
 access - provide land access to serve each end of the trip
 mobility - provide travel mobility at varying levels

There is a basic relationship between functionally classified highway


systems in serving traffic mobility and land access, as illustrated in the
next slide

 Arterials provide a high level of mobility and a greater degree of


access control, while local facilities provide a high level of access to
adjacent properties but a low level of mobility. Collector roadways
provide a balance between mobility and land access
272
Functional Classification
Arterials
•higher mobility
•low degree of access
Collectors
•balance between
mobility and access

Locals
•lower mobility
•high degree of access

Source: Safety Effectiveness of Highway Design


Features, Volume I, Access Control, FHWA, 1992

Access mobility
273
Functional Classification
 Conclusion:
The extent and degree of access control is a significant
factor in defining the functional class/category of the road

Freeway – accessibility lowest


mobility highest

What factors do we need to define LOS & class of road


1. Trip time – basic function
2. Operating speed
3. Absences of sudden changes of speed
4. Riding comfort

274
Functional Classification
Definition and characteristics of roads are based on
functional classification

Urban and rural functional systems are classified


separately since these areas differ in characteristics
with regard to:
1. Density and type of land use (L.U)
2. Density of population
3. Density of road network
4. Nature of travel pattern
5. The way in which these elements interact
275
Functional Classification
Urban area: in the USA urban areas are those places within
the boundaries set by responsible authority (officials) having
population > 5,000
Urbanized area > 50,000
Small urban area (urban clusters) 5,000 – 50,000

It is characterized by high population density

Forecast over population of the design year

Rural areas are places that fall outside the boundaries of


the urban area

276
Functional Classification
In addition to operational consideration, a road’s functional
classification influences several key design parameters
including:

Traffic volume that the road will carry


Traffic composition
Design & Operating speed
Design Standards
Type of trip

277
Functional Classification
Criteria to classify rural roads:
1. Trip length
2. Population size
3. Amount of traffic
4. Route spacing

278
Functional Classification
Criteria to classify urban roads:
1. Trip length
2. Population size
3. Amount of traffic
4. Route spacing
5. Access control

279
Hierarchies of Movement (HOM)
A road hierarchy has, for some time, been accepted as one
of the important tools used for road network and land use
planning

It is a means of defining each roadway in terms of its


function such that appropriate objectives for that roadway
can be set and appropriate design criteria can be
implemented

These objectives and design criteria are aimed at achieving


an efficient & safe road system whereby conflicts between
the roadway and the adjacent land use are minimized and the
appropriate level of interaction between the roadway and
land use is permitted 280
Hierarchies of Movement (HOM)
Transition from high speeds to lower speeds in a trip

Stages included in a trip


1. Main movement – on freeway
2. Transition – on freeway ramp to reduce speed
3. Distribution – on a moderate speed road
(Arterial/distributor)
4. Collection – on collector which passes through the
neighborhood
5. Access – on local road (access) that leads to individual
residences
6. Termination – as the vehicle is parked at the terminal
facility
281
Hierarchies of Movement (HOM)

Each element of the Functional Hierarchy can serve as a


collecting facility for the next higher element. However the
space and volume requirements for each element should be
defined in order to determine the cases where intermediate
element can be bypassed 282
Hierarchies of Movement (HOM)
Avoid sudden changes
Comments on HOM
1. HOM is based on the total amount of traffic
freeway
dist
collector
local
terminal
2. Intermediate stages are not always necessary
3. Complete stages of HOM relates specially to conditions of
low density suburban development where traffic flows are
cumulative on successive elements of the system

283
Hierarchies of Movement (HOM)
4. Conflict and congestion occur at the interface between
public roads and traffic generators where functional
transition is inadequate Shopping center

collector
Distributor
terminal Local

Freeway

5. Each functional category is related to a range of vehicle


speeds
6. The principles of HOM are related to the intensity of
traffic generator 284
Access Control
Access Control refers to the control or limitations on the
number of points of access to roads adjacent to property

Spacing & # of access points depends on speed/functional


classification of the road- the higher speed the longer spacing

Types of access control


Fully access control (when selected access points are
permitted)
Partially access control (when no direct access is
permitted)
No access control

285
Access Control
Accessibility

Mobility 286
Access Control
Notes:-
The functional conflicts lead to traffic congestion & safety
problems as the traffic mix changes to combine both through
traffic seeking to travel at higher speeds and low speed local
traffic turning in and out of local roads

Access should not be allowed at locations where entering


and leaving vehicles will create a hazard such as locations
where sight distance is limited or at point too close to an
intersection

287
Access Control
Types of access:
Roadside access

Median access/opening

Higher access lower capacity lower speed


lower LOS higher accident rates
288
Traffic Stream Performance (TSP)
Any factor that affects traffic stream performance
affects capacity as well

Factors affecting TSP:


1- geometric design features (H & V alignment,
grades, climbing lanes, sight distance, number of
lanes, width,…
2- traffic composition (HGV, LGV, PC, BUS)
3- road furniture (signs, signals, street lighting, traffic
control devices …)

289
Traffic Stream Performance (TSP)

4- type and condition of road pavement (road surface)


5- number and form of intersections
6- attractiveness of the road (sight seeing, terrain,..)
7- general environment (weather)
8- parking (bus stop)

290
Geometric Design Controls & Criteria
It relates to those characteristics of road user,
vehicle, and traffic that will affect the
improvement or optimization of the design of
various highway and street functional classes

Design parameters:
•Vehicle
•Road user (cover in traffic eng.)
•Traffic
•Speed
291
Vehicle Characteristics
The criteria of highway G.D is partly based on
the vehicle ch. (superelevation, widening)
We use the design vehicle

Power and Speed


Lighting
Performance (acceleration, breaking/deceleration)
Dimensions (Size & weight)
Wheels conditions
Instruments (seat belt)
292
Vehicle Characteristics
The physical characteristics of the vehicle in use and
the anticipated for future are essential in highway G.D

Dimensions (Size & weight)


G.D (lane & shoulder width, grads,
pavement design

Performance
Speed (G.D, S.D)
Power (resistance)
Acceleration
Deceleration 293
Vehicle Characteristics

Safety Standards
Instruments (air bags, seat belt)
Lighting (this affects night operation)

294
Design Vehicle
1- Design vehicle – the vehicle within a class that
represents all vehicles in that class – the worst in the
class
Design vehicles are selected motor vehicles with the
weight, dimensions, and operating characteristics
used to establish highway design controls for
accommodating all vehicles of that class and is
expected to use the road with considerable frequency
For the purposes of geometric design, each design
vehicle has larger physical dimensions and a larger
minimum turning radius than most vehicles in its
class. 295
Design Vehicle Dimensions

AASHTO,
Exhibit 2-1

296
Design Vehicle
The choice of design vehicle is influenced by:
Functional classification of a roadway
Proportions of the various types and sizes of
vehicles expected to use the facility
On rural facilities, to accommodate truck traffic,
one of the semitrailer combination trucks should be
considered in design.
In urban areas that are highly built-up, intersections
may be designed to provide fully for passenger
vehicles but require the larger vehicles to swing wide
upon turning. 297
Design Vehicle
The boundaries of the turning paths of the several design
vehicles when making the sharpest turns are established by
the outer trace of the front overhang and the path of the
inner rear wheel. (front left & rear right)

298
AASHTO Turning Templates

299
Driver Performance
The way how drivers interact with the design elements
& its information system and how they make errors
accidents and congestion
It is important to know the capabilities & limitations of
drivers which affects efficiency and safety of the roads
Errors occur when the design of the road is
incompatible with the drivers capabilities
Special consideration should be given to drivers > 65
1900 – 4%
1986 – (12 -16)%
2030 – 22%
Design driver !!!!
300
Driver Performance
Driving task:
Driving task can be complex under conditions such as
driving at high speed, poor environmental conditions,
where several individual activities need to be
performed simultaneously. Those requiring an
efficient & smooth integration and handling
information
The key for efficient and safe driving is free
information handling (without errors)

301
Driver Performance
 Causes of errors:
1. Inconsistent design
2. Deficiency of design
3. Unclear information
4. Complex situations
5. Pressure of time

 Levels of performance of activities


1. Control steering
2. Guidance road following Accidents
3. Navigation trip planning Inefficiency

Accidents302
Traffic Composition
For design of a particular hwy, data on traffic
composition should be determined by traffic
studies

Truck traffic should be expressed as a % of


total traffic during the design hour
 For a 2-lane hwy, as a % of total 2-way
traffic
 For multilane hwy, as a % of total traffic in
peak direction of travel
303
Traffic Composition
Criteria for determining traffic composition
under urban interrupted flow conditions
At important intersections, the % of trucks
during morning and evening peaks should be
determined separately
Variation in truck traffic bet. Int. movements
may be substantial

304
Volume / Future trends
Future trends/ projection of traffic demand:

Highways are designed to accommodate


traffic that is expected to occur within the life
expectancy of the highway- assuming adequate
maintenance

Life expectancy is difficult to find since each


segment is affected by different factors
305
Design Period
Life/design period:
R.O.W 100 yr
BASE COURSE 50 yr
DRIANAGE STRUCTURE 50 yr
BRIDGE (25 – 100) yr
SURFACING (10 -30) yr

What design period do we use?

306
Design Period
Design period (year) should be no further ahead than that
for which traffic can be estimated with reasonable degree of
accuracy
Use (15 – 25) yr
20 years design period is the most commonly used
We can't justify the prediction of volumes beyond this
period due to changes in regional economy, pop., land use,…
along the road which can’t be predicted with any degree of
assurance
For rehabilitation projects, 5 -10 years design period is used
due to uncertainty in traffic prediction and funding
constraints
307
Speed
Speed and Travel Time

Speed is the rate of motion in distance per


unite time
Travel time is the time taken to traverse a
defined section of the roadway
Speed and travel time are inversely related
S=d
t
308
Speed
Limitations introduced by speed
1. Reduce visual field…..Restricts peripheral vision
2. Limits time to receive & process information

How does the designer compensate for this


limitations
1. Aiding the driver by providing adequate
information
2. Placing the information in the cone of clear vision
3. Simplification of activities

309
Speed
1. Operating speed
2. Desired speed
3. Design speed
4. Running speed
1. Operating speed (OS): the speed at which drivers are
observed operating their vehicles during free-flow
conditions. The 85th percentile of the distribution of
observed speeds is most frequently used to measure the OS
associated with a particular location or geometric feature

 The 85th speed is generally determined from speed studies


made on a roadway after it has been opened to traffic 310
Speed
2. Posted speed: 2 popular methods
 10 mph under the design speed
 The use of 85th percentile speed/operating
speed

3. Desired speed: the operating speed that


drivers will adopt on less constrained
elements (large horizontal curves of
reasonably uniform section of road when not
constrained by other vehicles)
311
Speed
4. Design speed: the speed that is adopted for the calculation
of various G.D parameters (SD, superelevation, H curve radii)

Except for local streets where speed controls are frequently


included, every effort should be made to use as high design
speed as practical to attain a desired degree of safety,
mobility, and efficiency within the constraints of
environmental quality, economics and social or political
impacts

The design safe speed is usually equal to or greater than the


85th percentile speed/operating speed
312
Speed

Higher design speed – cost more, higher standard,


higher construction cost

What are the G.D features affected by the design


speed?
S.D
Curvatures
Superelevation
Friction

313
Speed
Guidelines for the selection of the design speed
1. Logical with regard to topography
2. Functional class of the road (driver’s expectation)
3. Adjacent land use (satisfy the travel desires &
habits of nearly all drivers)
4. Use as high speed as practicable to obtain a desire
degree of Safety, Efficiency & Mobility while under
Economics, Environmental consideration &
Social/political impact
5. Don’t use extremity high or low speed at which
small percentage use
314
Speed
6. Consider the average trip length
7. Use speed increments of 10 mph for low speed and of 5
mph at high speed. Each segment of the road is subjected to
different conditions leading to different speed

Design speed impacts most safety-related features of design


as well as those associated with rideability and comfort, it can
also impact efficiency & capacity of the roadway
(R, e, S.D, g are directly related to DS)

315
Design Speed (DS)

316
Highway Design Standards
Design speed
Depends on
Functional class of hwy
Topography
Highway location
Adjacent LU

317
Highway Design Standards
5. Running speed: speed of veh over
section of the road = distance traveled/
time veh in motion

318
Safety of Design
Safety of design
1. Full access control- the most significant measure to
improve safety
2. Speed
3. Use either control device or channelization at
intersections
4. Consider driver characteristics
5. Environmental impact of design
Measures to improve safety & efficiency of traffic
operations:-
•Reduce # of conflict points
•Signal phase for pedestrians
•Convert 2-way to one way
•Reduce turning & unnecessary cross walks 319
Basic Design Considerations
The geometric design of roads covers the design
features of roadway associated with safe, efficient and
comfortable travel

Basic design consideration includes:


1. Design speed
2. Sight distance
3. Horizontal alignment
4. Vertical alignment
5. Co-ordination of H & V alignments

320
Sight Distance (SD)
Sight distance: distance of a road ahead that is visible to
drivers

Assumptions:
1. Driver eye height
2. Object height
3. Oncoming vehicle height
4. Reaction time

321
Sight Distance (SD)

322
Sight Distance (SD)

Types of Sight Distances

 Stopping Sight Distance (SSD)


 Decision Sight Distance (DSD)
 Passing Sight Distance (PSD)
 Operational Sight Distance (OSD)

323
Sight Distance in Design
For safety, should provide sight distance of
sufficient length so that drivers can control the
operation of their vehicles to avoid striking an
unexpected object in the traveled way - STOPPING
SIGHT DISTANCE (SSD)

2-lane roads should have sufficient sight distance


to enable drivers to occupy the opposing traffic lane
for passing other vehicles without risk of crash -
PASSING SIGHT Distance (PSD)

324
Green Book (AASHTO) Policy Question

Sight distance assumes drivers are traveling at:


A. The posted speed limit
B. 10 mph above the speed limit
C. The 85% percentile spot speed of the
facility
D. The design speed of the facility

325
Stopping Sight Distance (SSD)
Stopping Sight Distance (SSD): the min sight
distance

It is the distance required by a below average driver


to stop his vehicle before reaching an obstacle in his
path

Note: The minimum designed stopping sight


distance should be long enough for a driver going at
design speed to see an object (potential hazard) and
stop before hitting the object
326
Stopping Sight Distance (SSD)
 Required for every point along
alignment (horizontal and
vertical) – Design for it, or sign
for lower, safe speed
 Available SSD = f(roadway
alignment, objects off the
alignment, object on road height
 SSD = PRD + BD (with final
SPEED = 0)

327
Minimum Required SSD
Recall that:
SSD = Reaction distance + Braking Distance dᵦ
Two components:
Distance traveled while reacting
(2.5 seconds assumed reaction time)
Distance traveled while braking
Assumes wet road (decel rate of 3.4 m/sec2 or
11.2 ft/sec2)
Can be calculated; however, minimum is usually
obtained by AASHTO book

328
Minimum Required SSD
Recall that:
SSD = Reaction distance + Braking Distance dᵦ

SSD = ST + Si² - Sf²


30(f+ G)
Where:
f= Friction Coefficient a/g
AASHTO recommends
a: deceleration rate 11.2 ft/sec²
g: acceleration due to gravity 32.2 ft/sec²
G= Grade; + for upgrade, - for downgrade
Si =initial speed, Sf = final speed (mph)
T: PRT
329
Sight Distance (SSD)
Example:
Use basic assumptions to determine SSD at 60 mph on
a) 0% grade, b) 3% grade

330
Minimum Design SSD; 2001 AASHTO

Source: A Policy on Geometric Design of Highways and Streets (The Green Book). Washington, DC.
American Association of State Highway and Transportation Officials, 2001 4th Ed. 331
Minimum Design SSD; 2001 AASHTO
Example:
Given: Available Sight distance = 430’ on a +3% grade
Find maximum speed if perception reaction time is
assumed to be 2.5 seconds

332
Minimum Design SSD; 2001 AASHTO

Consider analysis when vehicle skids across


different surfaces (a/g is not equal to 0.35)

Or final speed is not zero at the end of the skid,


as evidenced because the vehicle sustains
crushing damage until the vehicle is stopped.

333
Typical values for friction

Values of friction vary widely with road surface type, age, condition.
Surface type f (or a/g)
Concrete pavement -dry 0.60 to .75
Concrete pavement – wet 0.45 to .65
Asphalt pavement 0.55 to .70
Gravel 0.40 to .70
Ice 0.05 to .20
Snow 0.30 to .60

Source: Lynn Fricke, Northwestern Univ.

334
Decision Sight Distance (DSD)
SSD are sufficient to allow reasonably
capable and alert drivers to come to a quick
stop under ordinary circumstances

May be inadequate when drivers must make


complex or instantaneous decisions, when
information is difficult to perceive or when
unexpected or unusual maneuvers are
required
335
Decision Sight Distance (DSD)
Decision Sight Distance (DSD > SSD):
distance required for the driver to detect unexpected hazard or
unexpected information source, react to them and complete
safe maneuver safely and efficiently

 Requires higher P/R time


 Depends on type of maneuver made and roadway setting
(urban vs. rural)

It’s importance: give drivers additional margin for errors &


give them sufficient time to maneuver at a speed not to stop

336
Decision Sight Distance (DSD)

Source: A Policy on Geometric Design of Highways and Streets (The Green Book). Washington,
337
DC. American Association of State Highway and Transportation Officials, 2001 4th Ed.
Passing Sight Distance (PSD)
Passing Sight Distance (PSD > DSD > SSD):
distance needed to pass or overtake a slower vehicle
travelling in the same direction before meeting on
coming opposing vehicle

338
Passing Sight Distance / Assumptions
•The overtaken vehicle travels at a uniform speed

•The overtaking vehicle reduces speed and trails the


overtaken vehicle

•The overtaking vehicle drivers requires short time to react


before starting the passing maneuver

•The overtaking vehicle accelerates to a speed 10 mph higher


than overtaken vehicle’s speed during the overtaking
maneuver

•When the overtaking vehicle retains there should be suitable


clearance with opposing vehicle 339
Source: A
Policy on
Geometric
Design of
Highways and
Streets (The
Green Book).
Washington,
DC. American
Association of
State Highway
and
Transportation
Officials, 2001
4th Ed. 340
Passing Sight Distance
PSD = d1 + d2 + d3 + d4

d1 = distance traveled during P/R time to point


where vehicle just enters the left lane

d1 = 1.47t1(s – m + at1) …… PROOF !!!!


2
where
t1 = time for initial maneuver (sec)
s = average speed of passing vehicle (mph)
a = acceleration (mph/s)
m = difference between speeds of passing and
passed vehicle
341
Passing Sight Distance
d2 = distance traveled by vehicle while in left lane

d2 = 1.47st2
where:
s = speed of passing vehicle (mph)
t2 = time spent passing in left lane (sec)

d3 = clearance distance varies from 110 to 300 feet

d4 = distance traveled by opposing vehicle during passing


maneuver

d4 usually taken as 2/3 d2


342
Elements of Passing Sight Distance

343
Passing Sight Distance (PSD)
Design values for PSD

344
Passing Sight Distance (PSD)
Design values for PSD from MUTCD

345
Comparison between SSD & PSD

346
Operational Sight Distance OSD
Operational Sight Distance is not a
design consideration for divided highways.
OSD is based on the 85th percentile speed
at which 85% of traffic travels at or less.

347
Alignment Design

348
Outline

Concepts
1. Horizontal Alignment
a. Fundamentals
b. Superelevation

2. Vertical Alignment
a. Fundamentals
b. Crest Vertical Curves
c. Sag Vertical Curves
d. Examples

3. Other Non-Testable Stuff


349
Outline
• Alignment is a 3D problem broken down into two
2D problems
– Horizontal Alignment (plan view)
– Vertical Alignment (profile view)
• Stationing
– Along horizontal alignment
– 12+00 = 1,200 ft.

Piilani Highway on Maui

350
Road Design

• Examine the existing


topographical data.
Road Design
To create the vertical
profile of the road, lines
are projected from the
station points then the
vertical scale is created.
Elevation data from
existing station points are
plotted to a graph.

Usually expressed as a ratio


1V: 20H or as percentage
Road Design

A straight line is plotted through the


points. Vertical curves are added
between slopes. The station numbers
for PVC and PVT are computed.
Surveying and Stationing
 In order to define the location of a road, stations numbers
are used.
 Station: Start from an origin by stationing 0, regular stations
are established every 100 ft., and numbered 0+00, 12 + 00
(=1200 ft), 20 + 45 (2000 ft + 45) etc.
Station 1+50 would be 150 feet from the beginning of the
road.
Station 3+25 would be 325 feet from the beginning of the
road.
The distance from Sta. 1+07.84 to Sta. 6+75.26 is 567.41
feet.

354
Surveying and Stationing
Stationing goes from West to East for even-number
highways

Stationing goes from North to South for odd-number


highways

Ramps associated with even-number highways are stationed


from West to East.

Ramps associated with odd-number highways are stationed


from North to South

355
Horizontal Alignment

24
+0
23+00

0
22+00
21+00
20+00
19+00
18+00
17+0
plan

16

0
+0
0
15
+0
0

Looking at the PLAN VIEW, imagine looking straight


down on the project from a point directly above.
356
Vertical Alignment

The PROFILE VIEW is


like a SIDE VIEW as if
you are standing off to
one side of the road
and looking back at
the road.
profile 357
358
Curve Data
 Complete curve data must be calculated for every curve
on the road.
Complete curve data includes
the radius, central angle, length
of curve, and degree of
curvature.
All significant points (PI, PC,
and PT) must be identified on
the road plan.
The curve length is used to
find the station number of each
PC and PT.
Road Profile

NEED Permission
Road Charts

• Important data for the construction of the road


is contained in a road chart.
• Stations every 100 feet as well as all important
points are included on the chart.
Road Charts

Transit line for Project Road from Lead


BM Station
The Way Boulevard (N 90º W)
90.0’

0+00
78.68’ N3º37’W FROM BM to
Begin
centerline of Project Road
Project
P.C to the left
Curve Data
STA R = 125’
0+90.00 Δ=59º
L = 128.72
D=45.83º
Road Charts
Transit line for Short Cut Drive from
Station Project Road (Bearing N 25°3' E) to
Lead The Way Boulevard (N 0°4' W)

BM 0+00
78.68’ N3º37’W FROM BM to
90.0’

Begin
centerline of Project Road
Project

P.C to the left


Curve Data
STA R = 125’
0+90.00 Δ=59º
L = 128.72
D=45.83º
STA
P.T.
2+18.72
Road Charts
Continue adding information in this manner until you
arrive at the end of the road.
Transit line for Project Road from Lead The Way Boulevard (N 90º
Station
W)

0+00
Begin 78.68’ N3º37’W FROM BM to centerline of Project Road
Project

P.C to the left


Curve Data
STA R = 125’
0+90.00 Δ=59º
L = 128.72
D=45.83º

STA
P.T.
2+18.72

P.C. to the right


Curve Data
STA R = 200’
3+01.95 Δ=42º
L = 146.61
D = 26.65

STA
P.T.
4+48.56

STA
P.I. at Big Road
5+19.79
Alignment Design
 Iterative process

1. Estimated speed environment


2. Estimate design speed
3. Design elements
4. Determine speed environment
5. Compare with design speed

 Speed environment: it is the perception of the design


speed that drivers obtain from the overall alignment
standard, from the road section and terrain through which
they pass

365
Horizontal
Alignment
366
Horizontal Alignment
• Horizontal alignment consist of straight sections of the road, known as
tangent sections, connected by horizontal curves to ensure safety &
comfort

• The most obvious is the simple curve, which is just a standard curve
with a single, constant radius.

• Other options include;


– compound curve, which consists of two or more simple curves in
succession,
– and spiral curves which are continuously changing radius curves.

• The min radii of HC depends on:


– Design speed
– Superelevation
– Coefficient of lateral friction 367
Tangent Vs. Horizontal Curve
• Predicting speeds for tangent and horizontal
segments is different

• May actually be easier to predict speeds on curves


than tangents
– Speeds on curves are restricted to a few well defined
variables (e.g. radius, superelevation)
– Speeds on tangents are not as restricted by design
variables (e.g. driver attitude)
Horizontal Alignment
Desirable requirements for HC
 As direct as possible, consistent with topography
• Consistent (avoid sudden change in standards)
• Use visible curves outside the area of VC
• Avoid monitory design
 Use transition areas for sudden change in design
 Use genital curves (large R), always > min
 Coordinate between H & V alignments
 Bridges/ intersections should be on tangent
sections and clear of obstacles
369
Horizontal Alignment
Avoid
 Sudden change in design standards and speed
 Min radii
 Winding alignment (series of short curves)
 Compound / broken back curves
 Construction is difficult
 Drivers perception of the curve change

To solve this problem use tangent sections between


the HC
370
Layout of a Simple Horizontal Curve
Circular curve: it is a curve of constant radius connecting 2 tangents in
order to change direction
R = Radius of Circular Curve
BC = Beginning of Curve
(or PC = Point of Curvature)
EC = End of Curve
(or PT = Point of Tangency)
PI = Point of Intersection
T = Tangent Length
(T = PI – BC = EC - PI)
L = Length of Curvature
(L = EC – BC)
M = Middle Ordinate
E = External Distance
C = Chord Length
Δ = Deflection Angle
Degree of Curve
• It is the angle subtended by a 100-
ft arc along the horizontal curve.
• Is a measure of the sharpness of
curve and is frequently used
instead of the radius in the actual
construction of horizontal curve.
• The degree of curve is directly
related to the radius of the
horizontal curve by
Horizontal Curve Fundamentals

T  R tan
2 PI
T Δ
 100 E
L R 
180 D M
L
PC Δ/2 PT
 180 
100 
   18,000
D 
R  R R R

Δ/2 Δ/2
5729.6
D
R 373
Horizontal Curve Fundamentals
PI
T Δ
E

M
L
PC Δ/2 PT

 1 
E  R  1
 cos  2  R R

Δ/2 Δ/2
 
M  R1  cos 
 2 374
Example

A horizontal curve is designed with a 1500 ft. radius.


The tangent length is 400 ft. and the PT station is
20+00. What are the PI and PT stations?

375
Horizontal Curve
Rmin can be determined by several factors
1. Superelevation
2. SSD on HC when there is an obstacle inside the
curve
3. Coordination between the H & V alignments
such that to facilitate drainage
4. Appearance

376
Horizontal Alignment
• Along circular path, vehicle undergoes centripetal
acceleration towards center of curvature (lateral
acceleration)

• Balanced by superelevation and weight of vehicle


(friction between tire and roadway)

• Design based on appropriate relationship between


design speed and curvature and their relationship
with side friction and superelevation
Superelevation
Superelevation is the rotation of the road cross-
section in such a manner as to overcome the
centrifugal force acting on the vehicle traversing a
curve

378
Superelevation

Wp  Ff  Fcp
Rv

Fc ≈

e
W 1 ft

 WV 2  WV 2
W sin   f s W cos   sin    cos 
 gRv  gRv 379
Superelevation
• Figure illustrates the forces acting on a vehicle during
cornering. In this figure,  is the angle of inclination,
W is the weight of the vehicle in pounds with Wn and
Wp being the weight normal and parallel to the
roadway surface respectively.
• Ff is the side frictional force, Fc is the centrifugal force
with Fcp being the centrifugal force acting parallel to
the roadway surface, Fcn is the centrifugal force
acting normal to the roadway surface, and Rv is the
radius defined to the vehicle’s traveled path in ft.
Superelevation

 WV 2  WV 2
W sin   f s W cos   sin    cos 
 gRv  gRv
V2
tan   f s  1  f s tan  
gRv
V2
e  fs  1  f s e
gRv
2
V
Rv 
This is Rmin, emax & fmax g  f s  e 381
Superelevation
Factors to be considered in selecting (e):
1. Safety
2. Appearance
3. Drainage control
4. Longitudinal grade
5. Tendency to slide inward
6. Upper range of speed that drivers are likely to
drive on
7. Intersections / access points

382
Superelevation
• In actual design of a horizontal curve, the engineer
must select appropriate values of e and fs.
• Super-elevation value ‘e’ is critical since
– high rates of superelevation can cause vehicle steering
problems at exits on horizontal curves
– and in cold climates, ice on road ways can reduce fs and
vehicles are forced inwardly off the curve by gravitational
forces.
• Values of ‘e’ and ‘fs’ can be obtained from AASHTO
standards.
Superelevation
Ways to select (e):
1. emax , f any value < fmax
2. fmax , e any value < emax
3. e < emax, f < fmax

 The worst case is to use emax & fmax Rmin


 Design values of fs are chosen somewhat below
this maximum value so there is a margin of safety

384
Max e
 Controlled by:
– Climate conditions (amount of ice and snow)
– Terrain (flat, rolling, mountainous)
– Frequency of slow moving vehicles which are influenced by high
superelevation rates
(e max allowable) should not exceed:
• 0.12 :
• mountainous area without snow
• low volume gravel road to facilitate drainage

 0.10 : more common preferable


 Except for 3R projects – use 0.60 – 0.80:
• Resurfacing
• Restoring
• Rehabilitation 385
Superelevation
Use:
 0.60 for
• Urban without snow
• Mountainous experiencing heavy snow
• Short-term detour
 0.40 for
• Congested urban roads
• Intersections

386
Selection of e and fs
• Side friction factor (fs) variations
– Vehicle speed
– Pavement texture
– Tire condition
– Climate

387
Side Friction Factor

388
Side Friction Factor

389
Minimum Radius Tables

390
391
Design Superelevation Rates - AASHTO

392
from AASHTO’s A Policy on Geometric Design of Highways and Streets 2004
Example
A section of SR 522 is being designed as a high-speed
divided highway. The design speed is 70 mph. Using
WSDOT standards, what is the minimum curve radius
(as measured to the traveled vehicle path) for safe
vehicle operation?

393
Example
A curving roadway has a design speed of 110 km/hr. At one
horizontal curve, the
Super elevation has been set at 6.0% and the coefficient of
side friction is found to be 0.10. Determine the minimum
radius of the curve that will provide safe
2
V
Rv 
g ( f s  e)

R = 1102/9.8(0.10+0.06) = 595 meters

394
Example
Design radius example: assume a maximum e of 8%
and design speed of 60 mph, what is the
minimum radius?

fmax = 0.12 (from Green Book)


Rmin = _____602_________
15(0.08 + 0.12)

Rmin = 1200 feet

395
Example

For emax = 4%?

Rmin = _____602_________
15(0.04 + 0.12)
Rmin = 1,500 feet

396
Example
A horizontal curve with R = 800 ft is part of a 2-lane highway with a
posted speed limit of 35 mph. What is the minimum distance that a
large billboard can be placed from the centerline of the inside lane
of the curve without reducing required SSD? Assume b/r =2.5 sec
and a = 11.2 ft/sec2
SSD is given as
SSD = 1.47vt + _________v2____
30(__a___  G)
32.2
SSD = 1.47(35 mph)(2.5 sec) + _____(35 mph)2____
30(__11.2___  0)
32.2
= 246 feet

397
Example

m = R(1 – cos [28.65 S])


R
m = 800 (1 – cos [28.65 {246}]) = 9.43 feet
800

(in radians not degrees)

398
Example

• Deflection angle of a 4º curve is 55º25’, PC at


station 238 + 44.75. Find length of curve,T, and
station of PC.
• D = 4º
•  = 55º25’ = 55.417º
• D = _5729.58_ R = _5729.58_ = 1,432.4 ft
R 4

399
Example

• D = 4º
•  = 55.417º
• R = 1,432.4 ft
• L = 2R = 2(1,432.4 ft)(55.417º) = 1385.42ft
360 360

400
Example

• D = 4º
•  = 55.417º
• R = 1,432.4 ft
• L = 1385.42 ft
• T = R tan  = 1,432.4 ft tan (55.417) = 752.29 ft
2 2

401
Example
Stationing goes around horizontal curve.
For previous example, what is station of PT?
PC = 238 + 44.75
L = 1385.42 ft = 13 + 85.42
Station at PT = (238 + 44.75) + (13 + 85.42) = 252 + 30.17

402
Superelevation Transition
Superelevation runoff (ls): it is the general term denoting the length of
road required to accomplish the change in cross-slope from a section
with N.C to a fully super elevated section

(ls) should be uniform over a length adequate for likely operating


speed

(ls) depends on the rate at which the x-section is rotated

Used to:
•Add safety & comfort to the introduction of (e)
•Improve appearance ( gradual change)

Normal crown (N.C): mainly needed for drainage purposes


403
Superelevation Transition
Methods of obtaining e:
Rotation about the center line
Rotation about the inside edge
Rotation about the outside edge

The most commonly used & easiest is the rotation


around the center line

404
Superelevation Development

405
from the 2001 Caltrans Highway Design Manual
Superelevation Development

406
from AASHTO’s A Policy on Geometric Design of Highways and Streets 2001
Superelevation Runoff/Runout

407

from AASHTO’s A Policy on Geometric Design of Highways and Streets 2001


Superelevation
Road Section View Road Plan View

CL

2% 2%
Superelevation
Road Section View Road Plan View

CL

1.5% 2%
Superelevation
Road Section View Road Plan View

CL

1% 2%
Superelevation
Road Section View Road Plan View

0.5% CL 2%
Superelevation
Road Section View Road Plan View

CL 2%
-0.0%
Superelevation
Road Section View Road Plan View

CL
-0.5% 2%
Superelevation
Road Section View Road Plan View

CL
-1% 2%
Superelevation
Road Section View Road Plan View

-.5% 2%
CL
Superelevation
Road Section View Road Plan View

CL
-2% 2%
Superelevation
Road Section View Road Plan View

CL
-3% 3%
Super elevation
Road Section View Road Plan View

CL
-4%

4%
Superelevation
Road Section View Road Plan View

CL
-3% 3%
Superelevation
Road Section View Road Plan View

CL
-2% 2%
Superelevation
Road Section View Road Plan View

CL
-1.5% 2%
Superelevation
Road Section View Road Plan View

CL
-1% 2%
Superelevation
Road Section View Road Plan View

CL
-0.5% 2%
Superelevation
Road Section View Road Plan View

CL
-0.0% 2%
Superelevation
Road Section View Road Plan View

CL
0.5% 2%
Superelevation
Road Section View Road Plan View

CL
1% 2%
Superelevation
Road Section View Road Plan View

CL
1.5% 2%
Superelevation
Road Section View Road Plan View

CL
2% 2%
Suggested Steps on Horizontal Design
 Select tangents, PIs, and general curves make sure you
meet minimum radii

 Select specific curve radii calculate important points (see


lab) using formula or table (those needed for design,
plans

 Station alignment (as curves are encountered)

 Determine super and runoff for curves and put in table

 Add information to plans 429


Stopping Sight Distance on HC
 Min SD to provide along horizontal road is SSD
SSD
 100 s
SSD  Rv  s 
180 D
180SSD  Ms
s 
Rv
  90SSD 
M s  Rv 1  cos 
Obstruction

  Rv  Rv

Rv   Rv  M s 
cos  
1
SSD  Δs
90   Rv  430
Stopping Sight Distance on HC
• Adequate stopping sight distance must also be
provided in the design of horizontal curves.
• Sight distance restrictions on horizontal curves
occur when obstructions are present.
• Such obstructions are frequently encountered in
highway design due to the cost of right of way
acquisition and/or cost of moving earthen
materials.
• When such an obstruction exists, the stopping sight
distance is measured along the horizontal curve
from the center of the traveled lane.
431
Stopping Sight Distance on HC
• For a specified stopping sight distance, some
distance, Ms, must be visually cleared, so that the
line of sight is such that sufficient stopping sight
distance is available (min offset measured from the
CL of the inside lane
• Equations for computing SSD relationships for
horizontal curves can be derived by first
determining the central angle, s, for an arc equal
to the required stopping sight distance

432
Stopping Sight Distance on HC
• Assuming that the length of the horizontal
curve exceeds the required SSD, we have

100 s
SSD 
D
Combining the above equation with following
5729.6
D
R
we get; 57.296SSD
s 
Rv 433
Stopping Sight Distance on HC

• Rv is the radius to the vehicle’s traveled path,


which is also assumed to be the location of
the driver’s eye for sight distance, and is again
taken as the radius to the middle of the
innermost lane,
• and s is the angle subtended by an arc equal
to SSD in length.

434
Stopping Sight Distance on HC
• By substituting equation for s in equation of middle
ordinate, we get the following equation for middle
ordinate;
  28.65SSD  

Ms  Rv 1  cos  

  Rv  

• Where Ms is the middle ordinate necessary to provide


adequate stopping sight distance. Solving further we get;

Rv  1  Rv  M s 
SSD  cos  
28.65   R v 
435
Stopping Sight Distance on HC
Example:
A 6 degree curve (measured at the centerline of the inside lane) is
being designed for a highway
with a design speed of 70 mi/hr., the grade is level, the driver reaction
time is taken as 2.5 s
(ASSHTO’s standard value). What is the closest place that a roadside
object (trees etc) can be
Placed?
Solution: 2
70
d s  1.47 * 2.5 * 70   726.6 ft
30(0.348  0.01* 0)
The closest place of a object is given by:

5729.58 726.6 * 6
M [1  cos( )]  68.3 ft
6 200 436
Curve Types
 Simple circular curve

 Broken Back – two curves same direction (avoid)

 Compound curves: multiple curves connected directly
together (use with caution) go from large radii to smaller
radii and have R(large) < 1.5 R(small)

 Reverse curves – two curves, opposite direction (require


separation typically for superelevation attainment)

437
Compound

438
Broken Back
Two curves joined by
short straight line
Not desirable

439
Reverse Curve

440
HC Types Example
• Example:
• A horizontal curve is designed with a 2000-ft radius, the curve has a
tangent length of 400 ft and
• The PI is at station 103 + 00, determine the stationing of the PT.
• Solution: 
T  R tan
2

400  2000 tan
2
  22.62

L R  789.58 ft
180

PC  (103  00)  (4  00)  99  00

441
PT  PC  L  (99  00)  (7  89.58)  106  89.58
Spiral Curves

• No longer used
• Involve complex geometry
• Require more surveying
• Are somewhat empirical
• If used, superelevation transition should occur
entirely within spiral

442
Widening of Pavement on Curves
 Offtracting is a common ch to all vehs but much
more pronounced with larger vehicles

Reasons for widening on curves


1. A vehicle travelling on a curve segments of the
road occupies greater space than on a straight
segments
2. Vehicles tends to wander more on a curve
segments of the road occupies greater space than
on a straight segments
443
Widening of Pavement on Curves
Widening is affected by:
1. Radius of the H.C (R)
2. Width of pavement on the straight section (W)
3. The type & dimension of the design vehicle
4. Design speed
5. Traffic volume

Widening is costly compared to what is gained


A min value of 0.6 m (2 ft) is used

444
Widening of Pavement on Curves
Widening on circular curves is placed in the inside
of the curve

Widening should transition over the superelevation


runoff length but shorter lengths are sometimes used

445
Widening of Pavement on Curves

446
Widening of Pavement on Curves
WB-50

447
Widening of Pavement on Curves
Other DV

448
Vertical Alignment

449
Vertical Alignment
Consists of a series of grade tangents connected to
each other by parabolic V.C

May not be needed with the algebraic difference <


0.5%

450
Vertical Alignment
 Vertical alignment is mainly controlled by the following
factors
1. Functional classification of the road
2. Topography
3. Horizontal alignment
4. Safety
5. Drainage
6. Sight distance
7. Appearance
8. Design speed
9. Design vehicle characteristics
10. Traffic volume & composition
11. LU
451
Vertical Alignment
 In addition the following controls should be taken into
consideration:
1. Avoid short tangents between V.C
2. The length of V.C should be 2 -3 times the required
SSD
3. Coordinate between H & V alignments
4. Use smooth V.C consistent with the class of the road
5. Analyze downgrades for their effect on traffic
operation ( when steep down grades are unavoidable,
use emergency escape ramps)
6. Avoid broken back
452
Vertical Alignment
Grade (g) of a road is the rise/fall per unit of
horizontal distance along the road expressed as a
ratio or %
(g = ∆y/∆x)
It affects significantly the operating efficiency of
the vehicle
Trucks are mostly affected (g > 3%)
PC can navigate upgrades (4 -5) % without
significant loss in speed
453
TANGENT GRADES
Tangent grades are designated according to their slopes
or grades.
Maximum grades vary depending on the type of facility,
and usually do not have an absolute standard.
The effect of a steep grade is to slow down the heavier
vehicles (which typically have the lowest power/weight
ratios) and increase operating costs

454
Vertical Alignment
 Max grades
 Arterial 3%
 Local (12 – 16 )%
 Up to 20% under low traffic volume

 Min grades of 0.5% are used to meet drainage requirements


 On flat terrain level grades may be used, provided that
drainage can be adequately handled

Take into consideration:


1. Steepness of the grade
2. Length of the grade
3. Weight to horsepower of the vehicle
455
Vertical Alignment
• Objective:
– Determine elevation to ensure
• Proper drainage
• Acceptable level of safety
• Primary challenge
– Transition between two grades
– Vertical curves
Sag Vertical Curve
G1 G2
G1 G2
Crest Vertical Curve
Vertical Curve Fundamentals
• Parabolic function
– Constant rate of change of slope
– Implies equal curve tangents

y  ax  bx  c
2

• y is the roadway elevation x stations


(or feet) from the beginning of the curve
Vertical Curve Fundamentals

PVI
G1 δ
PVC G2
PVT
L/2

L
x

y  ax  bx  c
2
Choose Either:
• G1, G2 in decimal form, L in feet
• G1, G2 in percent, L in stations
Relationships

y  ax  bx  c
2

At thePVC : x  0 and Y  c
dY
At the PVC : x  0 and  b  G1
dx
d 2Y G2  G1 G2  G1
Anywhere: 2
 2a  a
dx L 2L
PVI
G1 δ
PVC G2
PVT
L/2

L
x
Example
A 400 ft. equal tangent crest vertical curve has a PVC station of 100+00 at 59
ft. elevation. The initial grade is 2.0 percent and the final grade is -4.5
percent. Determine the elevation and stationing of PVI, PVT, and the high
point of the curve.

PVI

PVT

PVC: STA 100+00


EL 59 ft.
Other Properties
G1 x

PVT
PVC

Y
Ym G2

PVI Yf
A  G1  G2

A 2 AL AL
Y x Ym  Yf 
200 L 800 200
•G1, G2 in percent
•L in feet
Vertical Curves
The min length of VC is governed by sight distance
consideration

For crest VC the daylight sight line controls the min


length of VC

For sag VC the sight distance is limited by the


headlamp range during nighttime driving conditions
Crest Vertical Curves
SSD

PVI
Line of Sight

PVC PVT G2
G1

h2
h1

For SSD < L For SSD > L

L
ASSD 
2

L  2SSD  

200 h1  h2 
2


100 2h1  2h2 
2
A
Crest Vertical Curves
• Assumptions for design
– h1 = driver’s eye height = 3.5 ft.
– h2 = tail light height = 2.0 ft.

• Simplified Equations
For SSD < L For SSD > L

ASSD 
2
L  2SSD  
2158
L
2158 A
Both equations yield equal results when SSD = L
Crest Vertical Curves
• Assuming L > SSD…

• K-Value (defines vertical curvature)


– The number of horizontal feet needed for a 1% change
in slope
SSD 2
K
2158

L
K
A
Design Controls for Crest Vertical Curves

from AASHTO’s A Policy on Geometric Design of Highways and Streets 2001


Design Controls for Crest Vertical Curves

from AASHTO’s A Policy on Geometric Design of Highways and Streets 2001


Sag Vertical Curves
Light Beam Distance (SSD)

G1
headlight beam (diverging from LOS by β degrees) G2

PVC PVT

h1 PVI
h2=0

L
For SSD < L For SSD > L

ASSD  200h1  SSD  tan  


2
L L  2SSD  
200h1  S tan   A
Sag Vertical Curves
• Assumptions for design
– h1 = headlight height = 2.0 ft.
– β = 1 degree

• Simplified Equations
For SSD < L For SSD > L

ASSD   400  3.5SSD  


2
L L  2SSD    
400  3.5SSD   A 
Both equations yield equal results when SSD = L
Sag Vertical Curves

• Assuming L > SSD…

2
SSD
K
400  3.5SSD

L
K
A
Design Controls for Sag Vertical Curves

from AASHTO’s A Policy on Geometric Design of Highways and Streets 2001


Design Controls for Sag Vertical Curves

from AASHTO’s A Policy on Geometric Design of Highways and Streets 2001


Example 1
A car is traveling at 30 mph in the country at night on a wet road
through a 150 ft. long sag vertical curve. The entering grade is -
2.4 percent and the exiting grade is 4.0 percent. A tree has
fallen across the road at approximately the PVT. Assuming the
driver cannot see the tree until it is lit by her headlights, is it
reasonable to expect the driver to be able to stop before hitting
the tree?
Example 2
Similar to Example 1 but for a crest curve.

A car is traveling at 30 mph in the country at night on a wet road


through a 150 ft. long crest vertical curve. The entering grade is
3.0 percent and the exiting grade is -3.4 percent. A tree has
fallen across the road at approximately the PVT. Is it
reasonable to expect the driver to be able to stop before hitting
the tree?
Example 3
A roadway is being designed using a 45 mph design speed.
One section of the roadway must go up and over a small hill
with an entering grade of 3.2 percent and an exiting grade of -
2.0 percent. How long must the vertical curve be?
Vertical Alignment/ Critical length
Critical length of grade (C.L):
The max length of designated upgrade on which a
loaded truck can operate without an unreasonable
reduction in speed
What is the common bases for determining C.L of
grades ?
The reduction in speed of truck below average
running speed

It is important b/c it affects safety 476


Vertical Alignment/ Climbing Lanes
 Climbing lane is an auxiliary lane in the upgrade direction
for use of heavy vehicles whose speed are significantly
reduced by the grade, provided for diverging of slow
vehicles from the through lanes thus allowing faster
vehicles to pass slower vehicle
 A 2-lane,2-way road + a C.L is not call a 3-lane road
 the provision of C.L on steep long gradient may be effective
in increasing capacity & reducing delay and most
importantly improve safety
 Hazard increase as gradient, length, traffic volume, percent
of trucks increase
 Signs & marking should encourage the drivers to use C.L
 C.L can be seen as a more effective alternative to reducing
gradient
 C.L are not used on multi-lane highways 477
Climbing Lanes

478
Climbing Lanes/ Warrants
Climbing lanes are considered “warranted” when
1)speed reduction and level of service—are
exceeded. Either warrant may be waived if, for
example, slow moving traffic is causing an identified
collision trend or congestion that could be corrected
by the addition of a climbing lane. However, under
most conditions,
2) heavy traffic volume (T) must exceed 45 heavy
vehicles on the grade in the design hour (i.e.
counting heavy vehicles travelling in both directions
on the grade). 479
Climbing Lanes/ Warrants
climbing lanes are built when both warrants are
satisfied

The AADT that would meet this warrant is


dependent on the design hour factor (k) and the %
of heavy vehicles however for example, if k = 0.15
and % heavy vehicles = 15%, then the design AADT
meeting this warrant would be 2000 (i.e. 2000 x
0.15 x 0.15 = 45)

480
Climbing Lanes / Geometric Features
The following geometric criteria should be met in
provision of climbing or passing lanes.
Lane Width
The width of the auxiliary lane should be the same
as the through lane
Shoulder Width
The shoulder adjacent to the auxiliary lane should
be equal to the standard shoulder width on that
design designation of highway
481
Climbing Lanes / Geometric Features
Superelevation
Superelevation on the climbing lane portion of the
roadway surface should generally be the same as
on the adjacent through lane. However, where
operating speeds of heavy vehicles can be expected
to be much lower than design speed, the designer
may use judgment in selecting a lower
superelevation rate

482
Climbing Lanes / Geometric Features
Tapers
The approach taper for entry to C.L should be
approximately 1H:20V while the taper to exist
should be 1H:50V
The full width of the C.L should begin at or before
the point where the speed of trucks is reduced by
10 km/hr

483
Climbing Lanes / Geometric Features
Proximity to Intersections
Locations that include or are in close proximity to
intersections should be avoided because of
possible operational difficulties. Where these
situations cannot be avoided, a site specific analysis
should be undertaken to determine the
intersection treatment required. The treatment
may require construction of an additional lane or
relocation of the intersection.

484
Climbing Lanes / Composite Grade
When a segment of a freeway consists of two or
more consecutive upgrades with different slopes,
the equivalent grade can be determined by using
one of two techniques:
1-Determine the average grade by finding the total
rise in elevation and dividing by the total horizontal
distance.
 this technique is valid where grades in all
subsections are < 4% or the total length of
composite grade is < 4000 ft
485
Climbing Lanes / Composite Grade
2- Estimate the value of equivalent continuous grade
GE that would result in the final speed for trucks
that is the same as that which would result from
the actual series of consecutive grades of the same
length ( using truck acc/dec curves)
 This technique should be used if any single
portion of the consecutive grades exceeds 4%
or if the total length of grades > 4000 ft

486
Climbing Lanes / Composite Grade
Truck acceleration/deceleration (performance)
curves are used based on vehicle with average
weight –to-horsepower ratio of 200Ib/hp

487
Climbing Lanes / Composite Grade

488
Emergency Escape Ramps
An emergency escape ramp is one provided on the
downgrade of a highway for the use of a driver who
lost control of the vehicle because of brake failure

The objective is to provide a lane that diverges


away from the main traffic stream while the
uncontrolled vehicle’s speed is gradually reduced
and the vehicle eventually brought to rest

489
Emergency Escape Ramps
The four basic most commonly types used for design

490
Vertical CURVE limited to provide
clearances
• Finally, vertical curve lengths may be limited
by the need to provide clearances over or
under objects such as overpasses or drainage
structures.

491
Coordination of Vertical and Horizontal
• Curvature and grade shouldAlignment
be in proper
balance
– Avoid
• Excessive curvature to achieve flat
grades
• Excessive grades to achieve flat
curvature
• Sharp horizontal curvature should not be
introduced at or near the top of a crest Image source:

vertical curve http://www.webs1.uidaho.edu/niatt_labmanual/Chapters/ge


ometricdesign/theoryandconcepts/DescendingGrades.htm

– Drivers may not perceive change in


horizontal alignment esp. at night Source: A Policy on Geometric
Design of Highways and Streets
(The Green Book). Washington, DC.
American Association of State
Highway and Transportation
Officials, 2001 4th Ed. P. 284
492
Coordination of Vertical and Horizontal
Alignment
• Sharp horizontal curvature should not be
introduced near bottom of steep grade near
the low point of a sag vertical curve
– Horizontal curves appear unclear
– Vehicle speeds (esp. trucks) are highest at the
bottom of a sag vertical curve Source: A Policy
on Geometric

– Can result in erratic motion Design of


Highways and
Streets (The
Green Book).
Washington, DC.
American
Association of
State Highway
and
Transportation
Officials, 2001 4th
Ed.
493
Coordination of Vertical and Horizontal
Alignment
• On two-lane roads when passing is allowed,
need to consider provision of passing lanes
– Difficult to accommodate with certain
arrangements of horizontal and vertical curvature
– need long tangent sections to assure sufficient
passing sight distance Source: A Policy
on Geometric
Design of
Highways and
Streets (The
Green Book).
Washington, DC.
American
Association of
State Highway
and
Transportation
Officials, 2001 4th
Ed.
494
Coordination of Vertical and Horizontal
Alignment
• At intersections where sight distance needs to be
accommodated, both horizontal and vertical curves
should be as flat as practical
• In residential areas, alignment should minimize
annoyance to neighborhood

Source: A Policy
on Geometric
Design of
Highways and
Streets (The
Green Book).
Washington, DC.
American
Association of
State Highway
and
Transportation
Officials, 2001 4th
Ed.
495
Coordination of Horizontal and Vertical
Alignment
•Coordination of horizontal and vertical alignment should begin
with preliminary design
•Easier to make adjustments at this stage
•Designer should study long, continuous stretches of
highway in both plan and
profile and visualize the
whole in three dimensions
(FHWA, Chapter 5)

496
Coordination of Horizontal and Vertical
Alignment

Source: FHWA,
Chapter 5 497
Coordination of Horizontal and Vertical
Alignment
• Should be consistent with the topography
• Preserve developed properties along the
road
• Incorporate community values
• Follow natural contours of the land

Source: FHWA,
Chapter 5 498
Good Coordination of Horizontal and
Vertical Alignment
• Does not affect
aesthetic, beautiful,
historic, and cultural
resources along the way
• Enhances attractive
beautiful views
– Rivers
– Rock formations
– Parks
– Historic sites
– Outstanding buildings
Source: FHWA,
Chapter 5
499
Source: A Policy
on Geometric
Design of
Highways and
Streets (The
Green Book).
Washington, DC.
American
Association of
State Highway
and
Transportation
Officials, 2001 4th
Ed. 500
There are 2 problems with this
alignment.
There are 2 problems with this alignment. What are they? Source: A Policy

What are they? on Geometric


Design of
Highways and
Streets (The
Green Book).
Washington, DC.
American
Association of
State Highway
and
Transportation
Officials, 2001 4th
Ed. 501
Source: A Policy
on Geometric
Design of
Highways and
Streets (The
Green Book).
Washington, DC.
American
Association of
State Highway
and
Transportation
Officials, 2001 4th
Ed. 502
Source: A Policy
on Geometric
Design of
Highways and
Streets (The
Green Book).
Washington, DC.
American
Association of
State Highway
and
Transportation
Officials, 2001 4th
Ed. 503
Source: A Policy
on Geometric
Design of
Highways and
Streets (The
Green Book).
Washington, DC.
American
Maybe we want this if we are trying to slow people down??? Association of
State Highway
and
Transportation
Officials, 2001 4th
Ed.
504
Source: A Policy
on Geometric
Design of
Highways and
Streets (The
Green Book).
Washington, DC.
American
Association of
State Highway
and
Transportation
Officials, 2001 4th
Ed.
505
Source: A Policy
on Geometric
Design of
Highways and
Streets (The
Green Book).
Washington, DC.
American
Association of
State Highway
and
Transportation
Officials, 2001 4th
Ed.
506
Source: A Policy
on Geometric
Design of
Highways and
Streets (The
Green Book).
Washington, DC.
American
Association of
State Highway
and
Transportation
Officials, 2001 4th
Ed.
507
Source: A Policy
on Geometric
Design of
Highways and
Streets (The
Green Book).
Washington, DC.
American
Association of
State Highway
and
Transportation
Officials, 2001 4th
Ed.
508
Source: A Policy
on Geometric
Design of
Highways and
Streets (The
Green Book).
Washington, DC.
American
Association of
State Highway
and
Transportation
Officials, 2001 4th
Ed.
509
Source: A Policy
on Geometric
Design of
Highways and
Streets (The
Green Book).
Washington, DC.
American
Association of
State Highway
and
Transportation
Officials, 2001 4th
Ed.
510
A

Source: A Policy
on Geometric
Design of
Highways and
Streets (The
Green Book).
Washington, DC.
American
Association of
State Highway
and
Transportation
Officials, 2001 4th
Ed.
511
Source: A Policy
on Geometric
Design of
Highways and
Streets (The
Green Book).
Washington, DC.
American
Association of
State Highway
and
Transportation
Officials, 2001 4th
Ed.
512
Source: A Policy
on Geometric
Design of
Highways and
Streets (The
Green Book).
Washington, DC.
American
Association of
State Highway
and
Transportation
Officials, 2001 4th
Ed.

513
GEOMETRIC CROSS SECTION
 The primary consideration in the design of cross sections is
drainage
 Highway cross sections consist of traveled way, shoulders (or
parking lanes), and drainage channels
 Shoulders are intended primarily as a safety feature
 Shoulders provide:
 accommodation of stopped vehicles
 emergency use,
 and lateral support of the pavement
 Shoulders may be either paved or unpaved
 Drainage channels may consist of ditches

514
Two-lane highway cross section, curbed.

Two-lane highway cross section, with ditches.

Two-lane highway cross section, curbed.


515
Divided highway cross section, depressed median, with ditches.

516
Divided highway cross section, raised median, curbed.

517
Geometric cross section cont..

• Standard lane widths are normally 3.6 m (12


ft), although narrower lanes are common on
older roadways, and may still be provided in
cases where the standard lane width is not
economical. Shoulders or parking lanes for
heavily traveled roads are normally 2.4 to 3.6
m (8 to 12 ft) in width; narrower shoulders are
sometimes used on lightly traveled road

518
Intersections

519
Definition
We can define the intersection as the
general area where two or more highways join
or cross ,including the roadway and roadside
facilities for traffic movements within the
area.

520
Intersections Categories
• Grade-separated without ramps

• Grade-separated with ramps

• At-grade

521
Grade-separated intersection
usually consist of structure that
provide for traffic to cross at
different levels ( vertical distance)
without interruption

522
Grade-separated type

523
Grade-separated type

524
At-grade intersections types
• T or three-leg intersections

• Cross intersections or four-leg

• Multileg intersections

525
At-grade intersections types

526
T intersections
• T intersections ranging from the simplest T
intersection to a channelized one with
divisional islands and turning roadways
• Channelization involves the provision of
facilities such as pavement markings and
traffic islands to regulate and direct conflicting

527
T intersections

528
Cross intersections or four-leg

529
Multileg intersections
• Have five or more approaches

• This type of intersection should be avoided in


order to remove some of the conflicting
movements from the major intersection and
increase safety and operation and that by
using realigned

530
Multileg intersections
When we realigned a road or convert it to a
road two factor must be consider :
• Should be realigned to major road

• The distance between the intersections should


be such that they can operate independently

531
Multileg intersections

532
Design principles for intersections
The fundamental objective in the design is
to minimize the severity or potential conflict
among traffic streams and between
pedestrians and turning vehicles

533
Design principles for intersections
the design of intersections involves:
• Alignment design
• A suitable channeling system
• Determine width of turning roadways for
traffic pattern
• Assurance of sight distance

534
Alignment of intersection
• The best alignment for the intersection when the
intersecting roads meet at right angle or nearly
right angle
• Why ???
1. Less required area for turning intersection
2. Lower exposure time for the vehicles crossing traffic
flow
3. Visibility limitations not serious as acute angle

535
Alignment of intersection

536
Profile of intersections
• A combination of grade lines should be
provided to facilitate the drivers control of
vehicle.
• Where possible large change in grade should
be avoided (not grater than 3%) , stopping &
acceleration distances (on grade 3% or less)
not much different from flat grade .

537
Profile of intersections
• When it’s unavoidable to use grade of 3% or
more design factors such stopping and
acceleration distance should be adjusted so
that conditions equivalent the those level
ground exist , in any case it is not advisable to
use grade higher 6 percent at intersections.

538
Profile of intersections
• When it is necessary to adjust the grade lines
of the approaches at an intersection, it is
preferable that the grade line of the major
highway be continued across the intersection
and that of the minor road be altered to
obtain the desired result (smooth junction and
proper drainage , clear traffic).

539
Intersections curves
• Main factor for curve design
1. The angle of turn
2. Turning speed
3. Design vehicle
4. Traffic volume

540
Turning speed
• Three types of design commonly used when turning
speeds are 15 mi/h or less
1. the simple curve (an arc of a circular curve)
2. the simple curve with taper
3. the three-centered compound curve (three simple
curves joined together and turning in the same
direction).

541
The simple curve (PC)
The inner radius of
edge pavement
(solid line) should
not be less than 25
ft, This design will
provide for a
clearance of
about 8" near the
end of the arc.
542
The simple curve with taper (PC)

this a simple curve


with tapers of 1:10
at each end and an
offset of 2.5 ft. In
this case, it is
feasible to use the
lower radius of 20
ft.

543
The three-centered compound curve (PC)

This type of curve is


composed of three
circular curves of radii
of 100, 20,and 100 ft,
with the center of the
middle curve located at
a distance of 22.5 ft,
including the 2.5 ft
offset, from the tangent
edges
544
The three-centered compound curve (PC)

This design is preferable to the simple


curve because it provides for a smoother
transition and because the resulting edge of
the pavement fits the design vehicle path
more closely.

545
Intersections curves
• In the included figures we have also the
minimum design for single-unit (SU) and tables for
minimum edge-of pavement for different angles of
turn and design vehicles

• When the turning speed at an intersection is greater


than 15 mi/h, the expected turning speed is used to
determine the minimum radius required

546
Intersection Channelization
• It’s defined as separation of conflicting traffic
movements into definite paths of travel by
traffic islands or pavement marking to
facilitate the safe and orderly movements of
path vehicles and pedestrians

547
Channelization
• Island :It’s defined area between traffic lanes
that used to regulate the movement of
vehicles or serve as pedestrian refuge.
• Good channelization increase the capacity ,
safety and driver confidence where the bad
one have the opposite effect and note that
overchannelization creates confusion

548
Channelization
• Factor effect channelization:
1. Availability of right of way
2. Terrain
3. Type of design vehicle
4. Cross sections of cross roads
5. Approach speed
6. Bus-stop requirement

549
Island
• It can formed by using:

1. Curbed traffic island

2. Pavement marking

3. Pavement edge

550
Island (curbed )
• Form by construction of a concrete curb that
delineates the area of island

• Classify as :
1. mountable
2. barrier

551
Island (curbed )
• Mountable curbs : constructed with their face
inclined at an angle 45 degrees or less
• Barrier : usually vertical

• Curbed island may be difficult to see at night


so it necessary to have fixed-source for
lighting

552
Island (pavement marking )
• Sometime referred to as flushed island
because flushed with the pavement
• Formed by marking that delineate the area of
the island
• Preferred over curbed islands at intersections
where approach speeds are relatively high and
pedestrian traffic low

553
Island (pavement edge )
• These islands are usually unpaved and are
mainly used at rural intersections where there
is space for large intersection curves

554
Island

555
Island
• It classify based on their function:
1. Channelized :mainly used to control and direct
traffic
2. Divisional : mainly used to divide opposing or
same-directional traffic streams
3. Refuge : used primarily to provide refuge for
pedestrians

556
Island

557
Island (Minimum size of island)
• AASHTO recommends that curbed islands have a minimum
area of approximately 50 sq ft for urban intersections ( 75 sq
ft for rural), although 100 sq ft is preferable for both
• The minimum side lengths recommended are 12 ft (but
preferably 15 ft) for triangular islands after the rounding of
corners, 20 to 25 ft for elongated or divisional islands, and 100
ft (but preferably several hundred feet) for curbed divisional
islands that are located at isolated intersections on high speed
highways.

558
Location of Approach Ends of Curbed
Islands
• The location of a curbed island at an intersection is
dictated by the edge of the through traffic lanes and
the turning roadways.
• The offset from the through traffic lane should be 2
to 3 ft, depending on factors such as the type of edge
treatment, island contrast, length of taper or
auxiliary pavement preceding the curbed island, and
traffic speed.

559
Location of Approach Ends of Curbed
Islands

560
Minimum Pavement Widths of Turning
Roadway Intersections
• Three classifications of pavement widths are used:
1. Case I: one-lane, one-way operation with no
provision for passing a stalled vehicle

2. Case II: one-lane, one-way operation with provision


for passing a stalled vehicle

3. Case III: two-lane operation, either one-way or two-


way

561
Minimum Pavement Widths of Turning
Roadway Intersections
• Case I is used mainly at relatively short connecting
roads with moderate turning volumes.
• Case II, which provides for the passing of a stalled
vehicle, is commonly used at locations where ramps
intersect with local roads and at channelized
intersections.
• Case III is used at one-way, high-volume locations
that require two-lanes or at two-way locations.

562
Minimum Pavement Widths of Turning
Roadway Intersections
• The pavement width depends:
1. Widths of the front and rear overhanges (FA,FB)
2. Total clearance per vehicle (C)
3. Extra width allowance due to difficulty of driving on
curves (Z)
4. Track width (U) of the vehicle as it moves around
the curve

563
Minimum Pavement Widths of Turning
Roadway Intersections

564
Minimum Pavement Widths of Turning
Roadway Intersections

565
Widths of the front and rear overhanges
(FA,FB)
• Value of FA can be obtained from this figure

566
Widths of the front and rear overhanges
(FA,FB)
• The width of the rear overhang FB is usually taken as
0.5 ft for passenger cars, since the width of the body
of a typical passenger car is 1 ft greater than the out-
to-out width of the rear wheels. For truck vehicles,
the rear overhang is 0 ft, since the width of truck
bodies is usually the same as the out to-out width of
the rear wheels.

567
Total clearance per vehicle (C)
• The pavement width provides for some lateral
clearance both between the edge of the
pavement and the nearest wheel path and
between the sides of the vehicles passing or
meeting.

568
Total clearance per vehicle (C)

• AASHTO suggests that the total clearance per


vehicle be taken as 4 ft for Case I, 2 ft for the
stopped vehicle and 2 ft for the passing
vehicle in Case II and 4 ft for Case III.

569
An extra width allowance (Z)
• It’s provided to compensate for the difficulty of maneuvering
on a curve and for variation in driver operation. It is obtained
from the empirical expression:

Where:
Z = extra width allowance to compensate for the difficulty in
maneuvering (ft)
v = design speed (mi/h)
R =radius of curve (ft)

570
Track width (U) of the vehicle
• The track width U for passenger cars and single-unit trucks is given as:

where
U =track width on curve (ft)
u =track width on tangent (out-to-out of tires) (ft)
R =radius of curve or turn (ft)
Li =wheelbase of design vehicle between consecutive axles or (sets of
tandem axles) and articulation points (ft)

571
Minimum Pavement Widths of Turning
Roadway Intersections
• The following table gives values for the
required pavement widths for different design
vehicles. In practice

572
Minimum Pavement Widths of Turning
Roadway Intersections

573
Minimum Pavement Widths of Turning
Roadway Intersections
• The design of the pavement width based on three types of traffic
conditions, each concerning a specific mix of vehicle types:

1. • Traffic Condition A: Passenger vehicles are predominant

2. • Traffic Condition B: Proportion of SU vehicles warrants this vehicle type


to be the design vehicle but it allows for the accommodation of some
tractor-trailer combination trucks (5 to 10%)

3. • Traffic Condition C: Proportion of tractor-semitrailer combination trucks


WB-12 or WB-15 (WB-40 or WB-50) vehicles in the traffic stream warrants
one of these vehicle types to be the design vehicle

574
Minimum Pavement Widths of Turning
Roadway Intersections

• This is table gives


pavement design widths
for the different
operational cases and
traffic conditions.

575
Sight distance at intersections
• it is necessary to provide an adequate view of the
crossroads or intersecting highways to reduce the
potential of collision with crossing vehicles.
• This requires an unobstructed triangular area (sight
triangle) that allows a clear view for drivers on the
minor and major roads to see an approaching vehicle
on the crossing road in time to avoid a potential
conflict

576
Sight distance at intersections

577
Sight Distance Obstruction

Hidden Vehicle

578
Sight distance at intersections
• There are two types of sight triangles,
approach sight triangles and departure sight
triangles.

579
Sight distance at intersections
• The approach sight triangle allows for the drivers on
both the major roads and minor roads to see
approaching intersecting vehicles in sufficient time to
avoid a potential collision by reducing the vehicle’s
speed or by stopping. The decision point on a minor
road of an uncontrolled or yield control intersection
is the location where the minor road driver should
start his/her braking or deceleration maneuver to
avoid a potential conflict with an approaching major
road vehicle.

580
Sight distance at intersections
• The departure-sight triangle allows for the
driver of a stopped vehicle on the minor road
to enter or cross the major road without
conflicting with an approaching vehicle from
either direction of the major road.

581
Sight distance at intersections
• the lengths of the legs of the sight triangle depend on the
type of control at the intersection.
1. At-grade intersections either have no control (Case A)
2. stop control on the minor road (Case B)
3. yield control on the minor road (Case C)
4. traffic signal control (Case D)
5. all-way stop control (Case E).
6. the maneuver of left turns from the major road (Case F).

582
NO control (Case A)
• Minimum sight triangle sides = distance traveled in
PRT and to actuate brake/accel.
• AASHTO recommend that PRT = 2.5 s also AASHTO
has noted that field observations have indicated
drivers tend to decrease their speeds to about 50
percent of their mid-block speed as they approach
intersections that have no control.

583
NO control (Case A)

584
NO control (Case A)

Critical speed is set to stopping distance db = a __da__


585
da - b
Example
Large
Tree 25 mph
72’

47’

45 mph

Is sufficient stopping sight distance


provided?

586
Example
Large 25 mph
Tree
b = 72’
db

a = 47’

50 mph da

db = a __da__
da - b

587
da = 220 feet

588
Example
Large 25 mph
Tree
b = 72’
db

a = 47’

45 mph da

da = 200 feet
db = a __da__ = 47’ (220’) = 69.9’
da – b 220’ – 72’
589
db = 69.9 feet
corresponds to 15 mph
590
Example
Large 25 mph
Tree
b = 72’
db

a = 47’

45 mph da

25 mph > 15 mph, stopping sight


distance is not sufficient for 25 mph

591
Stop control on the minor road (Case B)

Three Sub Cases


• Turn left on to major roadway (clear traffic
left, enter traffic right)
• Turn right on to major roadway (enter traffic
from left)
• Crossing (clear traffic left/right)

592
Stop control on the minor road (Case B)

593
Stop control on the minor road (Case B)

• The sight distance required for this maneuver


depends on the time the vehicle will take to
cross the intersection

594
Stop control on the minor road (Case B /
left turn)

595
Example
• A minor road intersects a major four-lane
undivided road with a design speed of 65
mi/h. The intersection is controlled with a stop
sign on the minor road. If the design vehicle is
a single-unit truck, determine the minimum
sight distance required on the major road that
will allow a stopped vehicle on the minor road
to safely turn
left if the approach grade on the minor road is
2%.

596
Example
• Solution:
• • Determine tg.
-From Table tg = 9.5 sec (for a single unit truck)
-Correct for number of lanes:
tg = (9.5 + 0.7) sec 10.2 sec
(Note no adjustment is necessary for approach grade
as it is not higher than 3%.)

• • Determine minimum sight distance

=1.47×65×10.2= 974.61 ft

597
Stop control on the minor road (Case B)

598
Stop control on the minor road (Case B /
right turn)
• similar to that for left turns discussed for Case
left turn, but the values of the (tg) are
adjusted in consideration of the fact that
drivers tend to accept gaps that are slightly
lower than those for left turns.
• AASHTO suggests that values in (tg) Table
should be decreased by 1 second.

599
Stop control on the minor road (Case B /
crossing)
• Minimum requirements determined for right
and left turns as presented for Cases left turn
and right turn will usually satisfy the
requirements for the crossing maneuver.

600
yield control on the minor road (Case C)

• The assumption made to determine the


minimum sight distance for this maneuver is
similar to Case A but with some modifications.

601
yield control on the minor road (Case C)

• Adequate sight distance on the major road


therefore should be provided for:
• crossing the intersection (Case C1)

• making right and left turns (Case C2).

602
crossing the intersection (Case C1)
• Drivers on minor roads approaching a yield sign tend
to decelerate to 60 percent of the minor road design
speed and not 50 percent
• The time tg to cross the intersection should include
the time taken for the vehicle to travel from the
decision point
• Cross and clear the intersection at the same speed

603
Crossing the intersection (Case C1)
• Based on previous assumptions

604
Crossing the intersection (Case C1)

605
Crossing the intersection (Case C1)

606
Example
An urban two-lane minor road crosses a four-lane divided
highway with a speed limit of 55 mi/h. If the minor road has a
speed limit of 35 mi/h and the intersection is controlled by a
yield sign on the minor road, determine the sight distance from
the intersection that is required along the major road such that
the driver of a vehicle on the minor road can safely cross the
intersection. The following conditions exist at the intersection.
• Major road lane width = 11 ft
• Median width = 8 ft
• Design vehicle on minor road is a passenger car length = 22 ft
• Approach grade on minor road =3%.

607
Example

• Solution: Use this Equ. to determine time (ta)


as the travel time to reach and clear the
intersection.
• tg = ta + (w + La)/0.88vmin
ta = 5.2 sec for passenger vehicles from Table
w = ( 4 × 11+ 8) = 52 ft
• tg = 5.2 + (52+ 22)/(0.88 × 35) = 7.6 sec

608
Example

dISD =1.47 × 55 × 7.6 = 614.5 ft

609
making right and left turns (Case C2)
• it is assumed that a driver will reduce his/her
speed to about 10 mph.
• Based on this assumption, the length of the
minor road leg of the sight triangle is taken as
82 ft.
• The length of the major road leg is computed
using the same principles for the stopped
control of Case B1 and B2.

610
traffic signal control (Case D)
• The two main requirements at signalized
intersections are:
1. the first vehicle stopped at the stop line of each
approach should be visible to the driver of the first
vehicle stopped on all other approaches
2. adequate sight distance should be provided for
left-turning vehicles to enable drivers of these
vehicles to select adequate gaps.

611
all-way stop control (Case E).
The only sight distance required in this case is
that the first vehicle stopped at the stop line
of each approach should be visible to the
driver of the first vehicle stopped on all other
approaches.

612
the maneuver of left turns from the major
road (Case F).
• the turning vehicle is assumed to start its
turning movement from a stopped position

• Adequate sight distance along the major road


is the distance traveled by an oncoming
vehicle along the major road during the time it
takes the stopped vehicle to cross the major

613
the maneuver of left turns from the major
road (Case F).

614
conclusion
• When we design the intersections
we need the safety elements as
possible to have smooth traffic with
no accident

615

You might also like